Sunteți pe pagina 1din 88

FIITJEE

MATHS
FIITJEE
PINNACLE
For – JEE (Main/Advanced)
FIITJEE Ltd. Material Provided by - Material Point Available on - Learnaf.com

PINNACLE-CBSE

MASTER ASSIGNMENT

CONTENTS

 True / False 1
 Fill in the Blanks 5
 Multi-choice (One correct
answer) 11
 Multi-choice (More than one
correct answer) 61
 Matching 70
 Comprehension 74
FIITJEE Ltd. Material Provided by - Material Point Available on - Learnaf.com

MASTER ASSIGNMENT
TRUE/ FALSE

State whether the following statements are True or False by writing / marking T or F.

1. If (x – r) is a factor of the polynomial f(x) = ax xn + ax – 1 xx – 1 +  + a1x+a0, repeated m


times (1 < m  n), then r is a root of f(x) = 0 repeated m times

2. The derivative of an even function is always an odd function.

3. If a < b < c < d, then the roots of the equation (x – a)(x – c) + 2(x – b)(x – d) = 0 are real
and distinct.

4. If the complex numbers Z1, Z2 and Z3 represent the vertices of an equilateral triangle
such that Z1  Z2  Z3 , then Z1 + Z2 + Z3 = 0.

5. If three complex numbers are in A. P. then they lie on a circle in the complex plane.

6. If P(x) = ax2 + bx + c, and Q(x) = – ax2 + dx + c when ac  0, then P(x) Q(x) = 0 has at
least two real roots.

7. The lines 2x + 3y = 19 and 9x + 6y – 17 = 0 cut the coordinate axes in concyclic points.


           
8.  
For any three vectors a,b and c , a  b  b  c  c  a  2a  b  c .     
 1
9. If  mi ,  , mi > 0, i = 1, 2 , 3, 4 are distinct points on any circle, then m1 m2 m3 m4 = 1.
 mi 

10. If A and B are two independent events and C is an event which defines that exactly one
of A and B occurs, then P(A  B) P(A  B) = P(C).

11. The equation 51x101 – 2323x100 – 45x + 1035 = 0 has at least one root in the interval
(451/100, 46).
   
12.   
If A a ,B b ,C c are three non-collinear points, then for any point P p in the plane of 
        

the triangle ABC, a b c   p  a  b  b  c  c  a . 
1 /2
tan1 x 1
13. The integral 
x
0
dx 
2  x cosec x dx
0

p q
 
14. For q > 0 and 
2
p ,
2 
0
cos x dx  q  sinp .

2n 1  i 2n
15. For all integral values of n, 2n
 n
 in  in .
1  i  2

FIITJEE Ltd., ICES House, 29-A, Kalu Sarai, Sarvapriya Vihar, New Delhi -110016, Ph 26515949, 26569493, Fax 26513942
FIITJEE Ltd. Material Provided by - Material Point Available on - Learnaf.com
M ast er A ssignm ent s P-57-P8-CBSE-MA-2

16. By eliminating x between the equations x2 + ax + b = 0 and xy + l(x + y) + al – b = 0, we


get a quadratic equation in y whose roots are the same as that of the given quadratic in x.

1  sin2x 1
17. lim is equal to .
x  / 4   4x 4

dy
18. The curve represented by  1  3x ln3 , y(2) = 7 cuts the x-axis at (1, 0).
dx

a b c 
If a, b, c are real positive numbers with abc = 1 and AA = 1 where A = b c a  , then
T
19.
c a b 
3 3 3
a + b + c = 4.

/2 /4
20. For an even function f,  f  cos2x  cos xdx  2  f sin2x  cos x dx .
0 0

21. An odd function f is defined as; f:[– 2a, a]  R and f(x) = f(2a – x) for x [– 2a, a]. If the
left hand derivative of f at x = a, is 0, then its left hand derivative at x = – a is also 0.

22. For a differentiable function f; f : [0, 4]  R, there exist some a, b  [0, 4], such that
(f(4))2 – (f(0))2 = 8f(a) f(b).

23. For a differentiable function f; f: [0, 4] R, there exist some ,   (0, 2) such that
4

 f  t  dt  f     f   .
2 2

24. If for all x  1, P(x) > P(x) and P(1) = 0, then P(x) > 0 for x > 1.

  
25. If b and c are any two non-collinear unit vectors and a is any vector, then
  
      a bc     
   
ab b  ac c   
bc
b  c  a.  

sin  cos  sin 2


 2   2   4 
26. For all values of , sin     cos     sin  2   = 0.
 3   3   3 
 2   2   4 
sin     cos    3  sin  2 
 3     3 

    2   2  
27. For any vectors u and v , u  v    u  v   u v.

FIITJEE Ltd., ICES House, 29-A, Kalu Sarai, Sarvapriya Vihar, New Delhi -110016, Ph 26515949, 26569493, Fax 26513942
FIITJEE Ltd. Material Provided by - Material Point Available on - Learnaf.com

P-57-P8-CBSE-MA-3 Pinnacle Study Package

28. If p is chosen at random from the interval [0, 5], then the probability of the equation
2 1 2
x + px +  p  2  = 0 having real roots is .
4 5

1 1
29. If for events A and B, P(A) = P  A / B  = and P(B/A) = , then A and B are mutually
4 2
exclusive.

30. If in a triangle ABC, a2 + b2 = 5c2, the medians to sides a and b are perpendicular to each
other.

FIITJEE Ltd., ICES House, 29-A, Kalu Sarai, Sarvapriya Vihar, New Delhi -110016, Ph 26515949, 26569493, Fax 26513942
FIITJEE Ltd. Material Provided by - Material Point Available on - Learnaf.com
M ast er A ssignm ent s P-57-P8-CBSE-MA-4

ANSWERS

1. F 2. T
3. T 4. T
5. F 6. T
7. T 8. F
9. T 10. F
11. T 12. T
13. T 14. F
15. T 16. T
17. F 18. T
19. F 20. F
21. T 22. T
23. F 24. T
25. T 26. T
27. F 28. F
29. F 30. T

FIITJEE Ltd., ICES House, 29-A, Kalu Sarai, Sarvapriya Vihar, New Delhi -110016, Ph 26515949, 26569493, Fax 26513942
FIITJEE Ltd. Material Provided by - Material Point Available on - Learnaf.com

P-57-P8-CBSE-MA-5 Pinnacle Study Package

Fill in the Blanks

Complete the following statements by filling the blanks with integers ranging from 0 to 9999.

1. The least value of ‘n’ such that (n  2)x2 + 8x + n + 4 > 0,  x  R, where n  N, is ….

2. If x2 + 3x + 5 = 0 and ax2 + bx + c = 0 have a common root and a, b, c  N, then


minimum value of a + b + c is equal to….
2
3. Total number of integral values of ‘a’ such that x + ax + a + 1 = 0 has integral roots is
equal to ….
2 2
4. The minimum value of 6x + 2y x  2xy 3y , where x, y  R, is equal to ………..
2
5. If x + 5 = 2x  4cos(a + bx), where a, b  (0, 5), is satisfied for atleast one real x, then
1
the maximum value of (a + b) is equal to ……….

6.  
For log7 log5 x  5  x = 0, x is equal to ……..

7. The number of solutions of log4(x  1) = log2(x  3) is …. …

5  9  13  ...   5y  1
8. If log2 x + log4x + log16x + … = y and  4log4 x then y is equal to
1  3  5  ...   2y  1
……

ba bc
9. If a, b, c are in H.P., then the value of  is equal to ….
ba bc

10. If z is a non-real root of (1)1/7, then (z86 + z175 + z289 ) is equal to ….

p q r a b c
11. If a, b, c, p, q, r are three complex numbers such that    1  i and    0 ,
a b c p q r
 p2 q2 r 2 
then the value of i  2  2  2  is equal to ……
a b c 

p b c
p q r
12. If for a  p, b  q, c  r, a q c  0 , the value of   is equal to ………
pa qb r c
a b r

13. Given 2x  y  2z = 2; x  2y + z = 4; x + y + z = 4, the value of  such that the given


system of equation has no solution, is ……….

5
1 
14. If the third term in the expansion of   xlog10 x  is 1000, then the value of x is ……..
x 

15. If the second, third and fourth terms in the expansion of (x + y)n are 135,30 and 10/3
respectively, then n is equal to ……….

FIITJEE Ltd., ICES House, 29-A, Kalu Sarai, Sarvapriya Vihar, New Delhi -110016, Ph 26515949, 26569493, Fax 26513942
FIITJEE Ltd. Material Provided by - Material Point Available on - Learnaf.com
M ast er A ssignm ent s P-57-P8-CBSE-MA-6

16. The greatest integer less than or equal to ( 2  1)6 is …..

17. If the sum of the coefficients in the expansion of (2x2  2x + 1)51 vanishes, then  is
equal to ……

18. The sides AB, BC CA of a triangle ABC have 3, 4 and 5 interior points respectively on
them. The total number of triangles that can be constructed by using these points as
vertices is ………..

19. Number of zeros at the end of 300! is equal to ……….

20. Total number of positive integral solutions of x1 + x2 + 2x3 = 15 is equal to ……….


 
21. The number of vectors of unit length perpendicular to vectors a  1, 1, 0  and b   0, 1, 1
is …………

 2   4 
22. If x sin = y sin      z sin    , then xy + yz + zx is equal to
 3   3 

23. The number of solutions of the equation sin3 = 4 sin sin(x + ) sin(x  ), (0 <  < )
in each quadrant, is ……….

a2  b2
24. In a triangle ABC, if C = 90, then sin (A – B) is equal to ……….
a2  b2

 1 1  2
25. If the line passing though P  ,  and with slope 1 intersects the curve 2x + xy  1
 2 2
1 1
= 0 at the points A and B, then  is equal to ………
PA PB

26. Number of points at unit distance from both the lines 3x  4y + 11 = 0 and 5x  12y + 7 =
0 is equal to ……….

27. If the circumference of the circle x2 + y2 – 2x + 8y – q = 0 is bisected by the circle


x2 + y2 + 4x + 22y + p = 0, then p + q is equal to ………
2 2
28 If (x, y) is a point lying on the circle x + y – 4x – 4y – 17 = 0, then the sum of the
2 2
maximum and minimum values of x + y + 3x + 4y is ……….
2 2
29. If r1 and r2 are distances of points on the ellipse 5x + 5y + 6xy  8 = 0 which are at
maximum and minimum distance from the origin, then r1 + r2 is equal to ……..

30. If f(x) = |x  1| |x  2| |x  3| and g(x) = (x  2)2, then number of solutions of f(x) = g(x) is
………

31. lim

tan sin2 x  is ………
x 0 1  cos x

32. For distinct a, b, c, d, if a, b are the roots of the equation x2  10x  11d = 0 and c, d
are the roots of the equation x2  10ax  11b = 0, then the value of a + b + c + d
is equal to ……… .

FIITJEE Ltd., ICES House, 29-A, Kalu Sarai, Sarvapriya Vihar, New Delhi -110016, Ph 26515949, 26569493, Fax 26513942
FIITJEE Ltd. Material Provided by - Material Point Available on - Learnaf.com

P-57-P8-CBSE-MA-7 Pinnacle Study Package

1
100

5050 1  x50
0
 dx
33. The value of 1
is ………
50 101
 1  x 
0
dx

2 3 n
3 3 3 n1  3 
34. If an =        ...   1   and bn = 1  an, then the minimum value of the
4  4  4 4
natural number n0, for which bn > an  n > n0, is ………

35. If f : R  R be a continuous function and f(x) = f(2x)  x  R with f(1) = 3, then the value
1
of  f  f  x   dx is equal to ………..
1

5
4
sin x  cos x
36. The value of 3
x

dx is equal to ………..
 e 4 1
4

2
37. For f(x) =  x  t dt ,
0
(x > 0), the minimum value of f(x) is …..

38. Let f(x) be a twice differentiable function such that f(x) =  f(x). If h(x) = (f(x))2 + f(x) = 5
at x = 5, then the value of h(10) is equal to ……

8
39. If the ordinate x = 2 2 divides the area, bounded by the x-axis, the curve y = 1+
x2
between the ordinates x = 2 and x = 4, into two parts whose areas are A1 and A2, then
A1A2 is equal to ………

40. The minimum area of the triangle formed by the x-axis, tangent and a normal at a point
on the curve y = x2 + 1 (1  x  3), is equal to ……….. square units.

41. The length of a double ordinate of the parabola y2 = 4x is varied at the rate proportional to
its length at any time. If the double ordinate is varied starting from the vertx for a length of
ln 4
time , where k is a constant of proportionality, the area tranversed is equal to ……….
3k
square units.

42. If a line makes the same angle  with the x-axis, the y-axis and the line x = y = z, then
  2
9  4 3 sec  is equal to …………

43. If two lines satisfying the relation  (l + m) = n, mn + nl + lm = 0 are perpendicular to


each other then the value of  is equal to ………..

44. If p is the length of the perpendicular from the origion to any member of the family of lines
c(a + b  2ab)x  2ab(y  5c) = 3abc, where a, b, c are in H.P., then the maximum value
2
of 2p is equal to ………

FIITJEE Ltd., ICES House, 29-A, Kalu Sarai, Sarvapriya Vihar, New Delhi -110016, Ph 26515949, 26569493, Fax 26513942
FIITJEE Ltd. Material Provided by - Material Point Available on - Learnaf.com
M ast er A ssignm ent s P-57-P8-CBSE-MA-8

 
sec  4  sec 2 
7 7
45. The value of is …………
 2 
 2  sec 
 7

1  sin 4A  1
46. If the equation y = has four solutions y1, y2, y3, y4, then y1 y2y3y4 is equal
1  sin 4A  1
to ………….

2
47. For any real x, the expression
a2
a  x  x  x 2  a2  can not exceed …….

48. If a, b, c are in A.P. and (1 + a), (2 + b), (1 + c) are in G.P., then the maximum value of ac
is ………….

49. John has x children by his first wife. Mary has (x + 1) children by her first husband. They
marry and have their own children. If the two children of the same parent do not fight, the
maximum number of fights between the children can be …………

50. If matrix A be such that A2 = A, then (1 + A)4 = 1 + kA where k is equal to ……

51. For two uni-modular complex numbers z1 and z2


1 1
 z1 z2   z1 z2  1/ 4 0 
z z1  z 
z1 
 k  where k is equal to ………..
 2  2  0 1/ 4 

3
52. The probabilty of occurance of event A and non-occurance of event B is , whereas
25
8
the probability of occurance of event B and non-occurance of event A is . If A and B
25
are independent events then 25 P(A) P(B) is equal to ……….

2
x 2 
 e t dt 
  
lim  x 
0
53. is eual to ………..
x 
2t 2
e
0
dt

54. If a square ABCD of diagonal length 7 3 is folded along AC, so that the planes DAC,
BAC are at right angles, the shortest distance between DC and AB is ………….

55. The line x + 2y + 3 = 0 cuts th circle x2 + y2 + 4x + 4y = 1 at P and Q, and the line 2x + 3y


+  = 0 cuts the circle x2 + y2 + 6x + 2y = 7 at R and S. If P, Q, R and S are concyclic then
 is equal to ………..

56. The angle between the tangents to the rectangular hyperbolas (y  mx) (mg  x) = a2
and (m2  1) (y2  x2) + 4mxy = b2 at their point of intersection is of ………. degrees.

FIITJEE Ltd., ICES House, 29-A, Kalu Sarai, Sarvapriya Vihar, New Delhi -110016, Ph 26515949, 26569493, Fax 26513942
FIITJEE Ltd. Material Provided by - Material Point Available on - Learnaf.com

P-57-P8-CBSE-MA-9 Pinnacle Study Package

a2  b2
57. If in a triangle ABC, A = 55, B = 15 and C = 110, then is equal to …………
ab

58. If the exression n sin2 + 2n cos( + ) sin sin + cos2( + ) is independent of , then
n is equal to …………

50 50 99.98 50
59. The value of 99  99. 98 + 97 + …. + 99 is ………..
1.2

A
a cot 
2
B
60. If for triangle ABC, b cot  = 0, then a(  ) + b(  ) + c(  ) is equal to
2
C
c cot 
2

FIITJEE Ltd., ICES House, 29-A, Kalu Sarai, Sarvapriya Vihar, New Delhi -110016, Ph 26515949, 26569493, Fax 26513942
FIITJEE Ltd. Material Provided by - Material Point Available on - Learnaf.com
M ast er A ssignm ent s P-57-P8-CBSE-MA-10

ANSWERS

1. 5 2. 9
3. 2 4. 11
5. 3 6. 4
7 1 8. 3
9. 2 10. 1
11. 2 12. 2
13. 1 14. 100
15. 5 16. 197
17. 1 18. 205
19. 74 20. 42
21. 2 22. 0
23. 1 24. 1
25. 6 26. 4
27. 50 28. 94
29. 5 30. 5
31. 2 32. 1210
33. 5051 34. 6
35. 6 36. 0
37. 1 38. 5
39. 2 40. 5
41. 4 42. 43
43. 2 44. 7
45. 4 46. 1
47. 2 48. 2
49. 191 50. 15
51 2 52. 12
53. 0 54. 7
55. 4 56. 90
57. 1 58. 2
59. 0 60. 0

FIITJEE Ltd., ICES House, 29-A, Kalu Sarai, Sarvapriya Vihar, New Delhi -110016, Ph 26515949, 26569493, Fax 26513942
FIITJEE Ltd. Material Provided by - Material Point Available on - Learnaf.com

P-57-P8-CBSE-MA-11 Pinnacle Study Package

MULTI-CHOICE
(One correct answer)
Indicate the correct choice by writing (marking) A, B, C or D.

1. The equation x2 + ax  a2  1 = 0 will have roots of opposite signs if


(A) a  (, ) (B) a  [1, 1]
(C) a  (, 1)  (1, ) (D) none of these
2
2. If sin, cos are the roots of ax + bx + c = 0, then
(A) a2 = b2 + 2ab (B) b2 = a2 + 2ab
2 2
(C) a = b + 2ac (D) b2 = a2 + 2ac

3. If x2 + 2ax + a < 0,  x  [1, 2], then


 4   4 1 
(A) a   ,  (B) a   , 
 5   5 3 
 1 
(C) a   ,  (D) none of these
 3 

4. Let a, b, c be positive real numbers forming a G.P. If ax 2 + 2bx + c = 0 and dx2 + 2ex
d e f
+ f = 0 have a common root, then , , are in
a b c
(A) G.P. (B) A.P.
(C) H.P. (D) none of these

5. If the equations x2 + ax + 12 = 0, x 2 + bx + 15 = 0 and x 2 + (a + b)x + 36 = 0 have a


common positive root, then the ordered pair (a, b) is
(A) (6, 8) (B) (7, 8)
(C) (6, 7) (D) (7, 9)

6. If ax2 + bx + 6 = 0 does not have two distinct real roots, a  R and b  R, then the least
value of 3a + b is
(A) 4 (B) 1
(C) 1 (D) 2
x
3
7. The solution set of    x  x 2  9 is
5
(A) {0} (B) {1}
(C)  (D) none of these

x x 1 x
8. The number of real roots of (sin2 ) (cos2 ) = (2 + 2x) is
4
(A) 0 (B) 1
(C) 2 (D) none of these

9. Let a, b, c be real numbers, a  0. If  is a root of a2x2 + bx + c = 0,  is a root of


a2x2  bx  2c = 0 and 0 <  < , then the equation a2x2 + 2bx + 2c = 0 has a root  that
always satisfies
  
(A)   (B)    
2 2
(C)  <  (D)  <  < 

FIITJEE Ltd., ICES House, 29-A, Kalu Sarai, Sarvapriya Vihar, New Delhi -110016, Ph 26515949, 26569493, Fax 26513942
FIITJEE Ltd. Material Provided by - Material Point Available on - Learnaf.com
M ast er A ssignm ent s P-57-P8-CBSE-MA-12

 
10. If ,  are the roots of x2 + px + q = 0 and also of x2n + pnxn + qn = 0 and if , are the
 
roots of xn + 1 + (x + 1)n = 0, then n is
(A) an odd integer (B) an even integer
(C) any integer (D) none of these
2 3
11. If x + px + 1 is a factor of ax + bx + c, then
2 2 2 2
(A) a + c = ab (B) a  c = ab
2 2
(C) a  c = ab (D) none of these
2 2 2 2 2 2
12. If (x + x + 2)  (a  3) (x + x + 1) (x + x + 2) + (a  4) (x + x + 1) = 0 has atleast one
real root, then complete set of values of ‘a’ is
 7 7 
(A)  1,  (B) 3 , 5
 3  
7   19 
(C)  , 1 (D)  5, 
3   3

13. If the quadratic equations 3x2 + ax + 1 = 0 and 2x2 + bx + 1= 0 have a common root, then
the value of 5ab  2a2  3b2, where a, b  R is equal to
(A) zero (B) 1
(C) 1 (D) none of these

14. Let a, b, c be non-zero real numbers such that


1 2

 (1  cos8 x) (ax 2  bx  c)dx  (1  cos8 x)(ax 2  bx  c)dx .



0 0
Then the quadratic equation ax2 + bx + c = 0 has
(A) no root in (0, 2) (B) atleast one root in (0, 2)
(C) a double root in (0, 2) (D) none of these
1
15. If x1 and x2 are the roots of x2 + (sin  1) x  cos2 = 0, then the maximum value of
2
x12 + x22 is equal to
(A) 2 (B) 3
9
(C) (D) 4
4
x
16. y = 10 is the refection of y = log10 x in the line
(A) y = 10x (B) 10y = x
(C) y = x (D) y =  x

17. If n is a natural number such that n  p11  p22  p33  pkk and p1, p2, …px are distinct
primes then ln n belongs to
(A) (k + ln2, ) (B) [k ln 2 , )
(C) (, k ln 2] (D) none of these

3 5
log2 x 2 log2 x 
18. The equation x 4 4  2 has

(A) exactly one real solutions


(B) exactly three real solutions
(C) exactly one rational root
(D) complex roots

FIITJEE Ltd., ICES House, 29-A, Kalu Sarai, Sarvapriya Vihar, New Delhi -110016, Ph 26515949, 26569493, Fax 26513942
FIITJEE Ltd. Material Provided by - Material Point Available on - Learnaf.com

P-57-P8-CBSE-MA-13 Pinnacle Study Package

19. The solution set of x > ½, logx2 log2x2 log24x > 1 belongs to
1
(A)  x  1 (B) 1 < x < 2
2
2
(C) 1 < x < 2 (D) none of these

n
r
20. r
r 1
4
 r2  1
is equal to

n2  n n2  2n
(A) (B)
2(n2  n  1) 2(n2  n  1)
2n2  n n2  n
(C) (D)
2(n2  n  1) n2  n  1
n
21.  r(n  r  1) is equal to
r 1
(A) n+1C3 (B) n+2C3
(C) n+1C2 (D) n+2C2

1 1 1
22. Value of 1      is equal to
1 2 1  2  3 1 2  3      n
2n 3n
(A) (B)
n1 2n  1
4n
(C) (D) none of these
3n  1

23. Sides of a triangle ABC; a, b, c are in G.P. If ‘r’ be the common ratio of this G.P., then
 5 1   5 1 
(A) r   ,  (B) r   , 
 2   2 
   
 5  1 5  1  5 1 5  3
(C) r   ,  (D) r   , 
 2 2   2 2 
 
24. If a, b, c are distinct positive real numbers such that a2 + b2 + c2 = 1, then ab + bc + ca is
(A) < 1 (B) > 1
(C) = 1 (D) none of these
n
r
25.  1.3.5.7.9...(2r  1) is equal to
r 1
1 1  1 1 
(A)  1   (B) 1  
2  1.3.5...(2n  1)  4  1.3.5...(2n  1) 
1 1 
(C)  1   (D) none of these
3  1.3.5...(2n  1) 

+ bc ac ab
26. If a, b, c  R , then   is always
bc ac ab
1 1
(A)  (a  b  c) (B)  abc
2 2
1 1
(C)  (a  b  c) (D)  abc
3 3

FIITJEE Ltd., ICES House, 29-A, Kalu Sarai, Sarvapriya Vihar, New Delhi -110016, Ph 26515949, 26569493, Fax 26513942
FIITJEE Ltd. Material Provided by - Material Point Available on - Learnaf.com
M ast er A ssignm ent s P-57-P8-CBSE-MA-14

27. If 4x2 + 9y2 + 16z2  6xy  12yz  8zx = 0, then x, y, z are in


(A) A.P. (B) G.P.
(C) H.P. (D) none of these

28. For a, b, c  R+, the minimum value of a(b2 + c2) + b(c2 + a2) + c(a2 + b2) is equal to
(A) abc (B) 2abc
(C) 3abc (D) none of these

2
4  e2
29. If f(x) = f(x) + f  t  dt , f(0) =
 , then the function f(x) is
3
0
2
e 1 e2  2
(A) e x  (B) 2e x 
3 3
e x e2  4
(C)  (D) none of these
3 3

30. If sides of a triangle are in A.P., and the greatest angle is two times the smallest angle,
then cosine of the least angle is
3 2
(A) (B)
8 3
1 3
(C) (D)
8 4

n1
31. If x1, x2, … xn are in H.P., then x
r 1
r  xr 1 is equal to

(A) (n  1) x1.xn (B) n.x1.xn


(C) (n + 1) x1.xn (D) none of these

n4
32. (1  x2 )2 (1  x)n  a x r
r
. If a0, a1, a2 are in A.P., then the value of ‘n’ can be
r 0
(A) n = 2 or 3 (B) n = 2 only
(C) n = 3 only (D) n = 5, n = 6

33. If a, b, c  R+ such that a + b + c = 18, then the maximum value of a2b3c4 is equal to
(A) 218.32 (B) 218.33
19 2
(C) 2 .3 (D) 219.33

1 3 7 15
34.        ‘n’ terms is equal to
2 4 8 16
n n
 1  1
(A) 2n  2    (B) n  2   
2  2
n
 1
(C) n  1    (D) none of these
2

  
2n 2n 2n  
35. If x   cos , y   sin , z   cos   sin2n  , where    0,  , then
n 0 n0 n 0  2
(A) xy = z (B) xyz = xy + z
(C) xyz = z + x (D) zy = x

FIITJEE Ltd., ICES House, 29-A, Kalu Sarai, Sarvapriya Vihar, New Delhi -110016, Ph 26515949, 26569493, Fax 26513942
FIITJEE Ltd. Material Provided by - Material Point Available on - Learnaf.com

P-57-P8-CBSE-MA-15 Pinnacle Study Package

36. If a, b, c  R+, and form an A.P. and abc = 4, then minimum value of ‘b’ is
(A) (2)2/3 (B) (2)1/3
2/3
(C) (4) (D) (5)4/3

37. If (p + q)th term of a G.P. is ‘a’ and its (p  q)th term is ‘b’, where a, b  R+, then its pth term
is
a3 b3
(A) (B)
b a
(C) ab (D) none of these

n
1
38.  log
r 1 2 r 4
is equal to

n(n  1) n(n  1)
(A) (B)
4 2
4
(C) n(n + 1) (D)
n(n  1)
n
r
39.  (r  1)! is equal to
r 1
1 1
(A) 1  (B) 1 
(n  1)! n!
1
(C) 1  (D) none of these
(n  1)!

 1 1  1  1  1 
40. Value of  1   1  2  1  4  1  8      1  n  is equal to
 3   3  3  3   32 
2n 1  2n
3  1 3  1 
(A)  1     (B)  1    
2 3  2 3 
   
2n 1 
3  1
(C)  1     (D) none of these
2 3 
 
2 2 2
41. If a, b, c are distinct real numbers such that a, b, c are in A.P. and a , b , c are in H.P.,
then
2 2
(A) 2b = ac (B) 4b = ac
2
(C) 2b = ac (D) 4b2 = ac

12 12  22 12  22  32
42. Value of       ‘n’ terms is equal to
1 1 2 1 2  3
2n2 n(n  2)
(A) (B)
n1 3
4n2
(C) (D) none of these
3n  2
2n
43. If x1, x2, … xn are in A.P., then  (1) r 1
 x r 2 is equal to
r 1

FIITJEE Ltd., ICES House, 29-A, Kalu Sarai, Sarvapriya Vihar, New Delhi -110016, Ph 26515949, 26569493, Fax 26513942
FIITJEE Ltd. Material Provided by - Material Point Available on - Learnaf.com
M ast er A ssignm ent s P-57-P8-CBSE-MA-16

n 2n
(A)
n 1

x12  x22n  (B)
2n  1
x12  x2n2
 
n n
(C)
2n  1

x12  x2n
2
 (D)
n1
x12  x 2n
2
 
44. If the equation ax2 + bx + c = 0, 0 < a < b < c, has non real complex roots z1 and z2, then
(A) |z1| > 1, |z2| < 1 (B) |z1| < 1, |z2| < 1
(C) |z1| < 1, |z2| > 1 (D) |z1| > 1, |z2| > 1

45. If |z1| = 1, |z2| = 2, |z3| = 3 and |9z1z2 + 4z1z3 + z2z3| = 12, then the value of |z1 + z2 + z3| is
equal to
(A) 2 (B) 3
(C) 4 (D) 6

46. If A(z1), B(z2), C(z3) are the vertices of an equilateral triangle ABC, the value of
 z  z3  2z1 
arg  2  is equal to
 z3  z 2 
 
(A) (B)
4 2
 
(C) (D)
3 6


47. The least value of p for which the two curves arg(z) = and z  2 3i  p intersect is
6
(A) 3 (B) 3
1 1
(C) (D)
3 3

6  6 
48. The argument of the complex number sin  i  1  cos  is
5  5 
6 5
(A) (B)
5 6
9 2
(C) (D)
10 5

 2 | z |2 2 | z | 3 
49. If logtan30    2 , then
 | z | 1
 
3 3
(A) | z |  (B) | z | 
2 2
(C) | z | > 2 (D) | z | < 2

50. If |z1  1| < 1, |z2  2| < 2, |z3  3| < 3, then |z1 + z2 + z3|
(A) is less than 6 (B) is more than 3
(C) is less than 12 (D) lies between 6 and 12

51. If 1, 1, 2, … , n1 are the n nth roots of unity and n is an even natural number, then
(1 + 1) (1 + 2) (1 + 3) … (1 + n1) equals
(A) 1 (B) 0
(C) 1 (D) none of these

FIITJEE Ltd., ICES House, 29-A, Kalu Sarai, Sarvapriya Vihar, New Delhi -110016, Ph 26515949, 26569493, Fax 26513942
FIITJEE Ltd. Material Provided by - Material Point Available on - Learnaf.com

P-57-P8-CBSE-MA-17 Pinnacle Study Package

52. Let z  1  t  i t 2  t  2 , where t is a real number. The locus of z in the argand plane is
(A) a hyperbola (B) an ellipse
(C) a straight line (D) none of these

53. For all complex numbers z1, z2 satisfying |z1| = 12 and |z2  3  4i| = 5, the maximum
value of |z1  z2| is
(A) 2 (B) 10
(C) 0.7 (D) 22
th
54. If z1 and z2 be the n roots of unity which subtend a right angle at the origin, then n must
be of the form
(A) 4k + 1 (B) 4k + 2
(C) 4k + 3 (D) 4k

55. Let z and  be two non-zero complex numbers such that | z | = |  | and Argz + Arg = .
Then z equals
(A)  (B)  
(C)   (D) 

56. If ( 1) is a cube root of unity, and (1 + )7 = A + B, then A and B are respectively
(A) 0, 1 (B) 1, 1
(C) 1, 0 (D) 1, 1
r r
57. If zr  cos 2
 isin , where r = 1, 2, 3, … n, then lim (z1.z 2 ... zn ) is equal to
n n2 n

 
(A) cos + i sin (B) sin  icos
2 2
(C) ei/2 (D) ei
58. The roots of the equation (z + )3 = 3,   0, represent the vertices of an equilateral
triangle of side
1
(A)  (B) 3 
3
1
(C) 3  (D) 
3
n n1
59. If z0 is one of the roots of the equation z cos0 + z cos1 + … + zcosn1 + cosn = 2,
where i  R, then
1 1
(A) z0  (B) z0 
2 2
1
(C) z0  (D) none of these
2
60. A(z1), B(z2), C(z3) are the vertices of a ABC inscribed in the circle | z | = 2. If internal
angle bisector of angle A meets the circum circle again at D(z4), then
2 z z
(A) z4 = z2z3 (B) z4  3 2
z1
zz zz
(C) z4  1 2 (D) z4  1 3
z3 z2
61. z1, z2 are the non-zero complex roots of z2  az + b = 0 such that |z1| = |z2|, where a, b
are complex numbers. If A and B represent the complex numbers z1 and z2 and
AOB = , ‘O’ being the origin, then

FIITJEE Ltd., ICES House, 29-A, Kalu Sarai, Sarvapriya Vihar, New Delhi -110016, Ph 26515949, 26569493, Fax 26513942
FIITJEE Ltd. Material Provided by - Material Point Available on - Learnaf.com
M ast er A ssignm ent s P-57-P8-CBSE-MA-18

 
(A) a2  4bcos2 (B) b2  4acos2
2 2
2 2  2 2 
(C) b  2acos (D) a  2bcos
2 2
 
62. If   is a square root of the two-rowed unit matrix, then  is equal to
  
(A)  (B) 
(C)  (D) none of these

63. If C is a skew-symmetric matrix of order n and X is n × 1 column matrix, then XCX is a


(A) scalar matrix (B) unit matrix
(C) null matrix (D) none of these

 1 2 1 x
64. If A    and f(x)  , then f(A) is
 2 1 1 x
1 1  1 1
(A)   (B)  
1 1  1 1
2 2
(C)   (D) none of these
2 2

65. If A is a skew-symmetric matrix of order 3, then A3 is


(A) skew-symmetric (B) symmetric
(C) diagonal (D) none of these

66. If P is a two-rowed matrix satisfying P = P1, then P can be


 cos   sin    cos  sin  
(A)   (B)  
  sin  cos     sin  cos  
  cos  sin  
(C)   (D) none of these
 sin   cos  

ax2 bx c
67. If, for y =    1,
 x  a  x  b  x  c   x  b  x  c  x  c
dy  a b c 
   f(x, y), then f(x, y) is
dx  a  x b  x c  a 
(A) xy (B) x + y
y x
(C) (D)
x y

 4 2i
68. For A    , (A  2I) (A  3I) is a
 i 1
(A) null matrix (B) Hermition matrix
(C) unit matrix (D) none of these

69. The differential equation representing the family of curves y2 = 2c(x + c ), c being a
parameter, is of
(A) order 1 (B) order 2
(C) degree 3 (D) degree 4

FIITJEE Ltd., ICES House, 29-A, Kalu Sarai, Sarvapriya Vihar, New Delhi -110016, Ph 26515949, 26569493, Fax 26513942
FIITJEE Ltd. Material Provided by - Material Point Available on - Learnaf.com

P-57-P8-CBSE-MA-19 Pinnacle Study Package

70. If square matrices A and B are such that AA = AA, BB = BB, AB = BA, then AB(AB)
is equal to
(A) ABAB (B) ABBA

(C) (AB) AB (D) none of these

71. Matrix A has x rows and x + 5 columns. Matrix B has y rows and 11  y columns. If both
AB and BA exist, then the ordered pair (x, y) is
(A) (3, 3) (B) (8, 3)
(C) (3, 8) (D) none of these
k 2 k1
72. If A = 0 (null matrix) for some value of k, then I + A + A + … + A is equal to
(A) 0 (B) (1  A)1
k
(C) (1 + A) (D) none of these
2 T
73. If S = I, where S is a skew symmetric matrix and I is a unit matrix, then SS is equal to
(A) I (B) I
(C) I + S (D) none of these

74. If A is the n × n matrix whose elements are all ‘1’ and B is the n × n matrix whose
diagonal elements are all ‘n’ and other elements are n  r, then A2 is a scalar multiple of
(A) B (B) A
(C) AB (D) none of these

75. If AB = A and BA = B, then A2 is equal to


(A) B2 (B) B
(C) A (D) none of these
a b c 
76. If abc = p, A  c a b  and a, b, c are the roots of the equation x3 + x2 + p = 0, then
b c a 
AAT is equal to
(A) I (B) I
(C) 0 (null) (D) none of these

77. If M is a 3 × 3 matrix, where det(M) = 1 and MMT = I, then det(M  I) is equal to


(A) 1 (B) 0
(C) 1 (D) none of these
 2  3
78. If A    and |A | = 125, then the value of  is
 2 
(A)  1 (B)  2
(C)  3 (D)  5
1 0 0  1 0 0
1
79. If A  0 1 1 , I  0 1 0  and A 1  (A 2
   cA  dI) , then the values of c and d are
6
0 2 4  0 0 1
(A) 6, 11 (B) 6, 11
(C) 6, 11 (D) 6, 11

 3 1 
 
80. If P   2 2  , A   1 0  and Q = PAPT, then PTQ2005P is equal to
 1  
3 0 1
 
 2 2 

FIITJEE Ltd., ICES House, 29-A, Kalu Sarai, Sarvapriya Vihar, New Delhi -110016, Ph 26515949, 26569493, Fax 26513942
FIITJEE Ltd. Material Provided by - Material Point Available on - Learnaf.com
M ast er A ssignm ent s P-57-P8-CBSE-MA-20

 1 2005   4  2005 3 6015 


(A)  (B)  
0 1   2005 4  2005 3 
1  2005 2  3
(C)   (D) none of these
4  2  3 2005 
x 3 7
81. Given that x = 9 is a root of 2 x 2  0 , the other two roots are
7 6 x
(A) 2, 3 (B) 2, 7
(C) 2, 5 (D) none of these

a b a  b
82. The determinant b c b  c is equal to zero, if
a  b b  c 0
(A) a, b, c are in A.P. (B) a, b, c are in G.P.
(C) a, b, c are in H.P. (D) one of these

sec x cos x sec 2 x  cot x cosecx /2


83. Let f(x)  cos2 x cos2 x cosec 2 x . Then  f(x)dx is equal to
2 2 0
1 cos x cos x
15 15   32
(A) (B)
60 60
15   32
(C)  (D) none of these
60


84. The values of  lying between  = 0 and  = and satisfying the equation
2
1  sin2  cos2  4 sin 4
2 2
sin  1  cos  4 sin 4  0 are
2 2
sin  cos  1  4 sin 4
7 5
(A) (B)
24 25

(C) (D) none of these
24

1 a a2  bc
85. The value of the determinant 1 b b2  ca is equal to
1 c c 2  ab
(A) abc (B) 0
(C) abc(a + b + c) (D) none of these

FIITJEE Ltd., ICES House, 29-A, Kalu Sarai, Sarvapriya Vihar, New Delhi -110016, Ph 26515949, 26569493, Fax 26513942
FIITJEE Ltd. Material Provided by - Material Point Available on - Learnaf.com

P-57-P8-CBSE-MA-21 Pinnacle Study Package

86. For positive numbers x, y, and z the numerical value of the determinant
1 logyx logzx
logxy 1 logzy is
logxz logzy 1
(a) 1 (B) 1
(C) 0 (D) none of these

87. The product of all the values of t, for which the system of equations (a  t)x + by + cz = 0;
bx + (c  t) y + az = 0; cx + ay + (b  t) z = 0 has non-trivial solutions, is
a c b a b c
(A) c b a (B) b c a
b a c c a b
a c b
(C) b a c (D) none of these
c b a

88. If the system of equations x + ay = 0, az + y = 0, ax + z = 0 has infinite solutions, then the


value of a is
(A) 1 (B) 1
(C) 0 (D) no real value

89. The value of C0 + 3C1 + 5C2 + … + (2n + 1) Cn is


(A) 2n.(n + 1) (B) 2n  1
2n
(C) Cn (D) none of these

90. The coefficient of xm in (1 + x)p + (1 + x)p+1 + … + (1 + x)n, p  m  n, is


(A) n+1Cm+1 (B) n1Cm1
n
(C) Cm (D) nCm+1

22 23 24 211
91. The value of 2  C0   C1   C2   C3      C10 is
2 2 4 11
(311  1) (211  1)
(A) (B)
11 11
(113  1) (112  1)
(C) (D)
11 11
100
92. The coefficient of x53 in the expansion  100
Cm (x  3)100 m  2m is
m 0
(A) 100C47 (B) 100C53
(C) 100C53 (D) 100C100
n n n n n
93. The coefficient of x in the polynomial (x + C0) (x + 3. C1) (X + 5. C2) … (x + (2n + 1) Cn)
is
n n+1
(A) n.2 (B) n.2
n n+2
(C) (n + 1).2 (D) n.2

94. Coefficient of x50 in (1 + x)1000 + 2x(1 + x)999 + 3x2(1 + x)998 + … + 1001x1000 is


(A) 1001C50 (B) 1000C50
1002
(C) C50 (D) 1002C51

FIITJEE Ltd., ICES House, 29-A, Kalu Sarai, Sarvapriya Vihar, New Delhi -110016, Ph 26515949, 26569493, Fax 26513942
FIITJEE Ltd. Material Provided by - Material Point Available on - Learnaf.com
M ast er A ssignm ent s P-57-P8-CBSE-MA-22

95.  
If R  5 5  11 and f = R  [R], where [ ] denotes the greatest integer function, then Rf
is equal to
(A) 22n+1 (B) 32n+1
2n+1 4n+1
(C) 4 (D) 2

1 10 2n 102 103 102n


96. The value of  C1   2nC2   2n
C3      is
81n 81n 81n 81n 81n
(A) 2 (B) 0
(C) 1/2 (D) 1

97. The coefficient of x4 in the expansion of (1 + x + x2 + x3)n is


(A) nC4 (B) nC4 + nC2
n n n n n n n n
(C) C4 + C1 + C4. C2 (D) C4 + C2 + C1. C2

98. If (1 + x)n = C0 + C1x + C2x2 + … + Cnxn, n being odd, then C12  C22  C23      Cn2 is equal
to
(A) 2n2 (B) 2n
2n! 2n!
(C) 2
(D)
2(n!) (n!)2

1 1 1
99. The value of       is equal to
n! 2!(n  2)! 4!(n  4)!
2n 2n2
(A) (B)
n! (n  1)!
2n1 2n1
(C) (D)
(n  2)! n!

100. Let (1 + x)n = C0 + C1x + C2x2 + … + Cnxn, and


C1 C C C 1
 2  2  3  3      n  n  n(n  1) . Then the value of k is
C0 C1 C2 Cn1 k
1
(A) (B) 2
2
1
(C) (D) 3
3
n 2 n
101. If (1 + x) = C0 + C1x + C2x + … + Cnx , then the value of
C0 + (C0 + C1) + (C0 + C1 + C2) + … + (C0 + C1 + C2 + … + Cn1) is
(A) n.2n1 (B) (n + 2)2n
n
(C) 2 (D) (n + 2)2n1
n
n
102. If x + y = 1, then r  Cr  xr  ynr equals
r 0
(A) 1 (B) n
(C) nx (D) ny
3
103. The coefficient of the term independent of x in the expansion of (1 + x + 2x )
9
3 2 1 
 x   is
2 3x 

FIITJEE Ltd., ICES House, 29-A, Kalu Sarai, Sarvapriya Vihar, New Delhi -110016, Ph 26515949, 26569493, Fax 26513942
FIITJEE Ltd. Material Provided by - Material Point Available on - Learnaf.com

P-57-P8-CBSE-MA-23 Pinnacle Study Package

1 19
(A) (B)
3 54
17 1
(C) (D)
54 4

50 50 50
104. If x = 99 + 100 , and y = (101) , then
(A) x = y (B) x < y
(C) x > y (D) none of these

105. The number of times the digit 5 will be written when listing the integers from 1 to 1000 is
(A) 256 (B) 280
(C) 300 (D) none of these

106. The number of ways in which an examiner can assign 30 marks to 8 questions, giving not
less than 2 marks to any question is
(A) 21C7 (B) 20 C7
(C) 19C7 (D) 18
C7

107. The number of triangles whose vertices are at the vertices of the octagon but none of
whose sides happen to come from the octagon is
(A) 16 (B) 18
(C) 36 (D) 70

108. The number of integers between 1 and 1000000 having the sum of the digits equal to 18,
is
(A) 33649 (B) 25927
(C) 41371 (D) none of these

109. The number of ways in which a mixed double game can be arranged from amongst 5
married couples if no husband and wife play in the same game is
(A) 56 (B) 60
(C) 96 (D) 129

110. In a chess tournament, all participants were to play one game with the other. Two players
fell ill after having played 3 games each. If total number of games played in the
tournament is equal to 84, then total number of participants in the beginning was equal to
(A) 10 (B) 14
(C) 12 (D) 15

111. Total number of ways of selecting two numbers from the set {1, 2, 3, 4, …, 3n} so that
their sum is divisible by 3 is equal to
3n2  n
(A) (B) 2n2  n
2
2n2  n
(C) (D) 3n2  n
2

112. Total numbers of regions in which ‘n’ coplanar lines can divide the plane, if it is known
that no two lines are parallel and no three of them are concurrent, is equal to
1 1

(A) n2  n  2
2
 (B)
2
3n2  n  

FIITJEE Ltd., ICES House, 29-A, Kalu Sarai, Sarvapriya Vihar, New Delhi -110016, Ph 26515949, 26569493, Fax 26513942
FIITJEE Ltd. Material Provided by - Material Point Available on - Learnaf.com
M ast er A ssignm ent s P-57-P8-CBSE-MA-24

1
(C)
2
3n  n2  
(D) n2  n  4 
113. The total number of ways of selecting 10 balls out of an unlimited number of identical
white, red and blue balls is equal to
(A) 12C2 (B) 12C3
(C) 10C2 (D) 10
C3

114. ‘n’ different toys have to be distributed among ‘n’ children. Total number of ways in which
these toys can be distributed so that exactly one child gets no toy, is equal to
(A) n! (B) n! nC2
(C)  n  1 ! nC2 (D) n! n1C2

115. Total number of words that can be formed using all letters of the word ’BRIZJESH’ that
neither begins with ‘I’ nor ends with ‘B’ is equal to
(A) 3720 (B) 3600
(C) 4920 (D) 4800

116 There are ‘n’ numbered seats around a round table. Total number of ways in which n1 (n1
< n) persons can sit around the round table, is equal to
(A) nCn1 (B) nPn1
(C) nCn1 (D) n
Pn1

117. Total number of ways in which 4 boys and 4 girls can be seated around a round table, so
that no two girls sit together, is equal to
(A) 7! (B) (3!)(4)!
(C) (4!)(4!) (D) (3!) (3!)

118. If letters of the word ‘KUMAR’ are written in all possible orders and arranged as in a
dictionary, then rank of the word ‘KUBER’ will be
(A) 67 (B) 68
(C) 69 (D) 65
119. The number of ways in which three distinct numbers in A.P. can be selected from the set
{1, 2, 3, …, 24} is equal to
(A) 132 (B) 198
(C) 66 (D) none of these
120. Two players P1 and P2 play a series of ‘2n’ games. Each game can result in either a win
or loss for P1. Total number of ways in which P1 can win the series of these games is
equal to
1 1 n

(A) 22n 2nCn
2
 (B)
2
2  2 2nCn  
1 1 2n

(C) 2n 2nCn
3
 (D)
3
2  2 2n Cn  
121. Total number of ways in which the letters of the word ‘MISSISSIPPI’ can be arranged, so
that any two S’s are separated, is equal to
(A) 7350 (B) 3650
(C) 6250 (D) 1261

122. Two persons are selected at random, from n persons seated in a row (n  3). The
probability that the selected persons are not seated consecutively, is equal to

FIITJEE Ltd., ICES House, 29-A, Kalu Sarai, Sarvapriya Vihar, New Delhi -110016, Ph 26515949, 26569493, Fax 26513942
FIITJEE Ltd. Material Provided by - Material Point Available on - Learnaf.com

P-57-P8-CBSE-MA-25 Pinnacle Study Package

n2 n 1
(A) (B)
n n
n2 n2
(C) (D)
n3 n 1

123. Three numbers are selected from the set {1, 2, 3, …, 23, 24}, without replacement. The
probability that the selected numbers form an A.P. is equal to
11 12
(A) (B)
23 23
3
(C) (D) none of these
46

124. A bag B1 has 3 white balls and 2 red balls. Another bag B2 has 4 white and 6 red balls. A
ball is drawn randomly from bag B1 and without seeing its colour, is put in bag B2. Now a
ball is drawn from bag B2. The probability of both the drawn balls, being of the same
colour, is
41 31
(A) (B)
55 55
29
(C) (D) none of these
55
125. In a bag, there are 15 red and 5 white balls. Two balls are drawn in succession, without
replacement. The first ball drawn is found to be red. The probability that second ball is
also red, is equal to
3 7
(A) (B)
10 10
5 14
(C) (D)
19 19
126. Two subsets A and B of a set S consisting of ‘n’ elements are constructed randomly. The
probability that A  B =  and A  B = 5 is equal to
n n
3 3
(A) 1    (B)  
4 4
1 1
(C) n (D) n
2 3
127. A natural number ‘n’ is selected at random from the set of first 100 natural numbers. The
100
probability that n   50 is equal to
n
9 39
(A) (B)
10 50
19 9
(C) (D)
20 20
1 1 1
128. Let ‘A’, ‘B’ and ‘C’ be three independent events with P(A) = , P(B) = and P(C) = .
3 2 4
The probability of exactly 2 of these events occurring, is equal to
1 7
(A) (B)
4 24
3 17
(C) (D)
4 24

FIITJEE Ltd., ICES House, 29-A, Kalu Sarai, Sarvapriya Vihar, New Delhi -110016, Ph 26515949, 26569493, Fax 26513942
FIITJEE Ltd. Material Provided by - Material Point Available on - Learnaf.com
M ast er A ssignm ent s P-57-P8-CBSE-MA-26

129. P1 and P2 throw a fair coin in turn. One who gets a head first, wins the game. If P1 starts
the game, probability that P2 wins the game, is equal to
2 1
(A) (B)
3 3
1 1
(C) (D)
2 4

130. A five digit number (having all different digits) is formed using the digits 1, 2, 3, 4, 5, 6, 7,
8 and 9. The probability that the formed number either begins or ends with an odd digit, is
equal to
5 1
(A) (B)
6 6
1 2
(C) (D)
3 3
131 A bag coutains 100 balls numbered from 1 to 100. Four balls are removed at random
without replacement. The probability that the number on the last ball is smaller than the
number on first ball, is equal to
1 3
(A) (B)
4 4
2 1
(C) (D)
5 2
132. 2n boys are randomly divided into two sub-groups containing n boys each. The
probability that the two tallest boys are in different groups is
n n 1
(A) (B)
2n  1 2n  1
2n  1
(C) (D) none of these
4n2

133. Two integers x and y are chosen with replacement out of the set {0, 1, 2, 3, …10} Then
the probability that x  y  5 is
81 30
(A) (B)
121 121
25 20
(C) (D)
121 121

134. If a coin is tossed m + n times, m > n, then the chance of getting atleast m consecutive
heads is
n2 m2
(A) m 1 (B) n 1
2 2
m2 n2
(C) m 1 (D) n 1
2 2

135. The unit vector perpendicular to the plane determined by P(1, 1, 2), Q(2, 0, 1) and R(0,
2, 1) is
1 1
(A)   2, 1, 1 (B)  1, 2, 1
6 6
1
(C) 
6
 1, 1, 2  (D) none of these

FIITJEE Ltd., ICES House, 29-A, Kalu Sarai, Sarvapriya Vihar, New Delhi -110016, Ph 26515949, 26569493, Fax 26513942
FIITJEE Ltd. Material Provided by - Material Point Available on - Learnaf.com

P-57-P8-CBSE-MA-27 Pinnacle Study Package

136. The area of the triangle with vertices A(1, 1, 2), B(2, 1, 1), C(3, 1, 2) is
(A) 13 (B) 13
(C) 21 (D) none of these

   
137. A, B, C and D are four points in a plane with position vectors a, b, c and d respectively
       
    
such that a  d  b  c  0  b  d   c  a  . For the triangle ABC, D is it’s
(A) centroid (B) orthocentre
(C) circumcentre (D) none of these

a a2 1  a3
 2
 2
 2
138. If b b2 1  b3  0 and the vectors A = (1, a, a ), B = (1, b, b ), C = (1, c, c ) are non-
c c 2 1  c3
coplanar, then the product abc is equal to
(A) 1 (B) 1
(C) 0 (D) none of these
     
139.
   A  A C B
If A, B and C are three non-coplanar vectors, then     
   A  C  is equal to
C A B C     A  B
(A) 2 (B) 2
(C) 0 (D) none of these
  
140. If A = (1, 1, 1) and C = (0, 1, 1) are given vectors, then a vectors B satisfying the
    
equations A  B  C and A  B  3 is
5 2 2  5 2 2
(A)  , ,  (B)   , , 
 3 3 3   3 3 3
5 2 2
(C)  , ,   (D) none of these
3 3 3

  
141. Let a  a1ˆi  a2 ˆj  a3kˆ , b  b1ˆi  b2 ˆj  b3kˆ and c  c1ˆi  c 2 ˆj  c 3kˆ be three non-zero
  
vectors such that c is a unit vector perpendicular to both the vectors a and b . If the
2
a1 a2 a3
  
angle between a and b is , then b1 b2 b3 is equal to
6
c1 c 2 c3
(A) 0 (B) 1
1 2 2 3 2 2 2
(C) a b (D) a b c
4 4

142. A vector a has components 2p and 1 with respect to a rectangular Cartesian system.
This system is rotated through a certain angle about the origin in the counter clockwise

sense. If a has components p + 1 and 1, then
1
(A) p = 0 (B) p = 1 or 
3
1
(C) p =  1 or (D) p = 1 or  1
3

FIITJEE Ltd., ICES House, 29-A, Kalu Sarai, Sarvapriya Vihar, New Delhi -110016, Ph 26515949, 26569493, Fax 26513942
FIITJEE Ltd. Material Provided by - Material Point Available on - Learnaf.com
M ast er A ssignm ent s P-57-P8-CBSE-MA-28

143. If the vectors a ˆi  ˆj  kˆ , ˆi  bjˆ  kˆ and ˆi  ˆj  ckˆ (a, b, c  1) are coplanar, then the value
1 1 1
of   is
1 a 1 b 1 c
(A) 1 (B) 1
(C) 0 (D) none of these
 
144. Let b  4iˆ  3 ˆj and c be two vectors perpendicular to each other in the xy-plane. A
 
vector in the same plane having projections 1 and 2 along b and c respectively is
(A) 2iˆ  ˆj (B) 2iˆ  ˆj
(C) ˆi  2ˆj (D) none of these

145. If x, y, z are real numbers and x2 + y2 + z2 = 5, then the maximum value of


(x  y)2 + (y  z)2 + (z  x)2 is
(A) 25 (B) 15
(C) 10 (D) 5
     
146. If a, b and a  b are unit vectors, then vectors a and b are inclined at an angle of
(A) 30 (B) 60
(C) 90 (D) 120

 
147. If x and y are two non-collinear vectors and ABC is a triangle with sides of length a, b, c
    
satisfying (a  b) x + (b  c) y + (c  a) x  y = 0 , then triangle ABC is
(A) acute angled (B) obtuse angled
(C) right angled (D) none of these
 ˆ , r   ˆi and r  ˆj , then
148. If a line makes the same angles , with the lines r  (iˆ  ˆj  k)
sec2 is equal to
(A) 9  4 3 (B) 4  9 3
(C) 9  4 3 (D) none of these

149. The origin O is the centre, and AB, AC are two diagonals of the adjacent faces of a
rectangular box. If angles AOB, BOC and COA are ,  and  respectively, then cos +
cos + cos is equal to
(A) 1 (B) 0
(C) 1 (D) none of these

x  4 y  2 z k
150. The value of k, such that   lies in the plane 2x  4y + z = 7, is
1 1 2
(A) 7 (B) 7
(C) 4 (D) none of these

151. A plane meets the coordinate axes at A, B, C such that the centroid of the triangle ABC is
at G(a, b, c). The equation of the plane ABC is
x y z x 4 z 1
(A)    1 (B)   
a b c a b c 3
x y z
(C)    3 (D) none of these
a b c

FIITJEE Ltd., ICES House, 29-A, Kalu Sarai, Sarvapriya Vihar, New Delhi -110016, Ph 26515949, 26569493, Fax 26513942
FIITJEE Ltd. Material Provided by - Material Point Available on - Learnaf.com

P-57-P8-CBSE-MA-29 Pinnacle Study Package

152. The direction cosines of two lines are related by l + m + n = 0 and al2 + bm2 + cx2 = 0.
The lines are parallel if
(A) a + b + c = 0 (B) a1 + b1 + c1 = 0
(C) a = b = c (D) none of these

153. Projection of the line x + y + z  3 = 0 = 2x + 3y + 4z  6 on the plane z = 0 is


x y6 z x y6 z
(A)   (B)  
2 1 0 1 2 0
x y6 z
(C)   (D) none of these
1 2 0

154. The plane x  2y + 3z = 17 divides the line joining the points (2, 4, 7) and (3, 5, 8) in
the ratio
(A) 3 : 5 (B) 3 : 10
(C) 3 : 7 (D) none of these

155. The distance of the point (1, 2, 3) from the plane x  y + z = 5, measured parallel to the
x y z
line   , is
2 3 6
1
(A) (B) 1
7
(C) 7 (D) none of these

156. (l1, m1, n1), (l2, m2, n2), (l3, m3, n3) are the direction cosines of three mutually
perpendicular lines. If the line, whose direction ratios are l1+ l2+ l3, m1+ m2+ m3, n1+
n2+n3, makes angle  with any of these three lines, then cos is equal to
1 1
(A) (B)
2 3
1
(C) (D) none of these
3

157. A plane at a constant distance p from the origin meets the coordinate axes in A, B, C.
Locus of the centroid of the triangle ABC is
(A) x2 + y2 + z2 = p2 (B) x2 + y2 + z2 = 9p2
1 1 1 1
(C) x + y + z = 3p (D) none of these

158. The equations to the straight line through (a, b, c) parallel to the z-axis are
x a y b z c
(A)   (B) z = c, x = y
1 1 0
(C) x = a, y = b (D) none of these

159. A spherical rain drop evaporates at a rate proportional to its surface area at any instant t.
The differential equation governing the rate of change of the radius r of the rain drop is
(for some constant )
dr dr
(A)  r (B)  r 2
dt dt
dr
(C)   (C) none of these
dt

FIITJEE Ltd., ICES House, 29-A, Kalu Sarai, Sarvapriya Vihar, New Delhi -110016, Ph 26515949, 26569493, Fax 26513942
FIITJEE Ltd. Material Provided by - Material Point Available on - Learnaf.com
M ast er A ssignm ent s P-57-P8-CBSE-MA-30

x 1 y 1 z 1 x 3 y k z
160. If the lines   and   intersect, then the value of k is
2 3 4 1 2 1
1 2
(A) (B)
3 3
1
(C)  (D) 1
3

161. If a sec = 1  b tan and a2sec2 = 5 + b2 tan2, then


2 2 2 2 2 2 2 2
(A) a b  4a = 9b (B) a b + 4a = 9b
2 2 2 2
(C) a b + 9b = 4a (D) 4a b + 9b = 5a2
2 2 2

162. For how many values of x between 0 and 2 is the equation 2cosec 2x cotx  cot2x = 1
valid ?
(A) 0 (B) 2
(C) 1 (D) none of these

163. If m tan(  30°) = n tan( + 120°), then cos2 =


(m  n) (m  n)
(A) (B)
2(m  n) 2(m  n)
(m  n) (m  n)
(C) (D)
(m  n) (m  n)

164. If A = cos2 + cos2( + )  2cos cos × cos( + ), then


(A) A > 1 (B) 0  A  1
(C) 1  A < 0 (D) A <  1

1 x
165. If tan   , tan   and tan = (x3 + x2 + x1)1/2, then for x > 0
2 2
x(x  x  1) x  x 1
(A)  +  = 2 (B)  +  = 3
(C)  +  =  (D) 2( + ) = 

3  cot 76 cot16


166. The value of 
cot 76  cot16
(A) cot 44° (B) tan 44°
(C) cot 46° (D) tan 56°

167. If angle A of triangle ABC is given by the equation 5cosA + 3 = 0, then sinA and tanA are
the roots of the equation
(A) 8x2 + 15x  16 = 0 (B) 15x2 + 8x  16 = 0
2
(C) 16x + 15x  8 = 0 (D) none of these

168. If cos(x  y) + cos(y  z) + cos(z  x) = 3/2 and cosx + cosy + cosz = a, sinx + siny +
sinz = b, then
(A) a = 1, b = 0 (B) a = 0, b = 1
(C) a = 1, b = 1 (D) a = 0, b = 0

169. If 1, 2, 3 and 4 be four values of  which satisfy the equation 3tan 3 = tan(45° + ),
the value of tan1 + tan2 + tan3 + tan4 is equal to
(A) 1 (B) 2
(C) 1 (D) 0

FIITJEE Ltd., ICES House, 29-A, Kalu Sarai, Sarvapriya Vihar, New Delhi -110016, Ph 26515949, 26569493, Fax 26513942
FIITJEE Ltd. Material Provided by - Material Point Available on - Learnaf.com

P-57-P8-CBSE-MA-31 Pinnacle Study Package

170. If A, B, C be the angles of a triangle ABC, such that A = /4 and tanB tanC = , then

(A)   ( 2  1)2 , ( 2  1)2  (B)   (1, 2)
(C)   ( 2  1) or   ( 2  1)2
2
(D) none of these

ab
171. If tan tan = , then (a  b cos2) (a  b cos2) is
ab
(a) independent of  (B) independent of 
(C) independent of  and  (D) none of these

172. In any acute angled triangle ABC, if P = tanA tanB tanC, then
(A) P  3 3 (B) P  3 3
(C) P  2 3 (D) none of these

173. If angle C of a triangle ABC be obtuse, then


(A) tanA tanB < 1 (B) tanA tanB > 1
(C) tanA tanB = 1 (D) none of these

174. In a triangle ABC, sin3A cos(B  C) + sin3B cos(C  A) + sin3C cos(A  B) is equal to
(A) 3cosA cosB cosC (B) 3sinA sinB sinC
(C) 2cosA cosB cosC (D) 2sinA sinB sinC

175. If A = 580°, then


A 1 A
(A) sin     1  sin A  1  sin A  (B) sin      1  sin A  1  sin A 
2 2 2  
A 1 A
(C) sin      1  sin A  1  sin A  (D) cos    1  sin A  1  cos A
2 2 2

3
176. If cos(A  B) = and tanA tanB = 2, then
5
2 2
(A) cosA cosB = (B) sinA sinB = 
5 5
1
(C) cos(A + B) =  (D) none of these
5

177. The values of ‘a’ for which the equation sin4x + cos4x = a has real solutions belong to
1
(A) 0  a  1 (B)  a  1
2
1
(C)  a  1 (D) none of these
4

178. If   n and tan is the geometric mean between sin and cos, then 2  4 sin2 + 3sin4
 sin6 is equal to
(A) 1 (B) 0
(C) 1 (D0 2

179. The equation cos2x + a sinx = 2a  7 possesses a solution if


(A) a < 2 (B) 2  a  6
(C) a > 6 (D) a is any integer

FIITJEE Ltd., ICES House, 29-A, Kalu Sarai, Sarvapriya Vihar, New Delhi -110016, Ph 26515949, 26569493, Fax 26513942
FIITJEE Ltd. Material Provided by - Material Point Available on - Learnaf.com
M ast er A ssignm ent s P-57-P8-CBSE-MA-32

180. The number of solutions of sin2x + cos2x + cosx + 1 = 0 in the first quadrant is
(A) 0 (B) 1
(C) 2 (D) infinite

2 2
181. The smallest positive root of the equation 81sin x  81cos x  30 is
 
(A) (B)
3 4

(C) (D) none of these
12

 1  1  1  1  1 
182. The value of sin1  
 2  2 
1
  cos     tan  3  cot  
 3

 is 
5 17
(A) (B)
12 12
(C) 0 (D) 1

183. If 1 < x < 2 , then number of solutions of the equation


tan1(x  1) + tan1x + tan1(x + 1) = tan13x is
(A) 0 (B) 1
(C) 2 (D) 3
184. The inequality sin-1(sin5) > x2  4x holds if
(A) x = 2  9  2 (B) x = 2 + 9  2

(C) x  2  9  2, 2  9  2  (D) x > 2  2  9

185. The number of solutions of the equation


3(sinx + cosx)  2(sin3x + cos3x) = 8 is
(A) 0 (B) 1
(C) 2 (D) infinite

186. If A = tan1 and B = tan11 then


(A) A = B (B) A > B
(C) A < B (D) none of these

187. If S = cos12 + cos18 + cos118 + cot132 + …  then S =


 
(A) (B)
2 3
 
(C) (D)
4 6

188. The value of the determinant


1 a a2
cos  n  1 x cosnx cos n  1 x is zero if
sin  n  1 x sinnx sin n  1 x
(A) a = 0 (B) cosx = 0
(C) sinx = 0 (D) none of these

189. The solution of the equation


tan + tan4 + tan7 = tan tan4 tan7 is

FIITJEE Ltd., ICES House, 29-A, Kalu Sarai, Sarvapriya Vihar, New Delhi -110016, Ph 26515949, 26569493, Fax 26513942
FIITJEE Ltd. Material Provided by - Material Point Available on - Learnaf.com

P-57-P8-CBSE-MA-33 Pinnacle Study Package

n n
(A) , nI (B) , nI
12 4
n n
(C) , n  I (D) , nI
3 6
6 6
190. If the equation sin x + cos x =  has real solutions, then the range of  is
 1 1 
(A)  ,   1,   (B)  , 1
 4  4 
1  1 
(C)  , 1 (D)  , 1
3  2 

191. Values of  for which the equation sin4x + cos4x + sin2x +  = 0 is valid are
 5 3
(A) (1, 1) (B)   , 
 2 2
 1   2 1
(C)   , 1 (D)   , 
 2   2 2

192. The values of k for which the system of equations


sinx cos2y = (k2  1)2 + 1 and
cosx sin2y = k + 1
has a solution, are
(A) (1, 1) (B) {1}
(C) {1, 1} (D) {1}

193. If cos1p + cos1q + cos1r =  (0  p, q, r  1), then the value of p2 + q2 + r2 + 2pqr is


(A) 0 (B) 1
(C) 2 (D) none of these

194. If A = sin[cot1{cos(tan1x)}], then


1  x2 2  x2
(A) A = (B) A =
2  x2 4  x2
2  x2 3  x2
(C) A = (D) A =
3  x2 7  x2

2x 3x  x 2
195. tan1 x  tan1    tan1 , (x > 0) is true if
1 x2 1  3x 2
1 1
(A) x < (B) <x<1
3 3
1 1
(C) x > (D) <x< 3
3 3

  7   7  
196. The value of sin  tan1  tan   cos 1  cos   is
  6   3 
(A) 0 (B) 1
(C) 1 (D) none of these

 4 2
197. The numerical value of tan  cos1  tan1  is
 5 3

FIITJEE Ltd., ICES House, 29-A, Kalu Sarai, Sarvapriya Vihar, New Delhi -110016, Ph 26515949, 26569493, Fax 26513942
FIITJEE Ltd. Material Provided by - Material Point Available on - Learnaf.com
M ast er A ssignm ent s P-57-P8-CBSE-MA-34

6 17
(A) (B)
17 6
15 6
(C) (D)
6 15


198. Solution of the equation sin16x + sin16 3 x =  is
2
1 1
(A) (B) 
12 12
1
(C)  (D) none of these
12
  1 
199. Value of sin  tan1 x  tan1    is
  x 
(A) 1 for x > 0,  1 for x < 0 (B) 1 for x < 0,  1 for x > 0
(C) 1, for all x  0 (D) 1, for all x  0

200. The general solution of the equation


sin4x + cos4x = sinx cosx is

(A)
 2n  1  , n  I (B)
 6n  1 
, n I
2 6

(C)
 4n  1 
, n I (D) n, n  I
4

201. In a triangle ABC, if r1 = 2r2 = 3r3, then a : b is equal to


(A) 1 : 2 (B) 3 : 4
(C) 5 : 4 (D) 2 : 3

a b c
202. If for a triangle ABC, b c a  0 , then sin3 A + sin3 B + sin3 C is equal to
c a b
(A) 3 sin A sin B cos C (B) 3 sin A sin B sin C
(C) 2 cos A cos B cos C (D) 3 cos A cos B cos C

203. If in any triangle r = r1 – r2 – r3, then the triangle is


(A) Right angled (B) Isosceles
(C) Equilateral (C) none of these

204. In a triangle ABC, if tan (A/2) = 5/6 and tan (B/2) = 20/37, the sides a, b and c are in
(A) G.P. (B) H.P.
(C) A.P. (D) none of these

205. In a ABC, a : b : c = 4 : 5 : 6. The ratio of the radius of the circumcircle to that of the
incircle is
(A) 16 / 7 (B) 7 / 16
(C) 15 / 7 (C) 7 / 15

206. In a  ABC, B =  / 8 and C = 5 / 8 and the altitude AD = h. Then h : a is equal to


(A) 2 : 1 (B) 3 : 1
(C) 1 : 3 (D) 1 : 2

FIITJEE Ltd., ICES House, 29-A, Kalu Sarai, Sarvapriya Vihar, New Delhi -110016, Ph 26515949, 26569493, Fax 26513942
FIITJEE Ltd. Material Provided by - Material Point Available on - Learnaf.com

P-57-P8-CBSE-MA-35 Pinnacle Study Package

207. In the ABC, A > B. If the measures of A and B satisfy the equation 3 sin x – 4sin2 x – k =
0, 0 < k < 1, then the measure of C is
(A)  / 3 (B)  / 2
(C) 2 / 3 (D) 5 / 6

208. If in a ABC, b : c = 2 : 1 and sin (B – C) = 3 / 5 then the ABC is


(A) Right angled (B) obtuse angled
(C) isosceles (D) none of these

209. In a right angled triangle the hypotenuse is four times as long as the distance of the
hypotenuse from the opposite vertex. Its acute angles are
(A) 30, 60  (B) 15, 75
(C) 45, 45 (D) none of these
210. In a right angled triangle ABC, the bisector of the right angle C divides AB into segments
A B
p and q, and tan  t . Then
2
(A) p : q = (1 + t) : (1 – t) (B) p : q = (1 – t) : (1 + t)
(C) p : q = 1 : 2 (D) p : q = 2 : 1

211. In a triangle ABC, C = 60 and A = 75. If D is the point on AC such that the area of
the BAD is 3 times the area of the BCD, then ABD is
(A) 30 (B) 60
(C) 90 (D) 45

212. If in a ABC, A = 45, B = 75, then


(A) a  c 3  2b (B) a + c = 2b
(C) a  c 2  2b (D) a – c = 2b

213. If the median of ABC through A is perpendicular to AB then tan A + 2 tan B is equal to
(A) 2 (B) 1
(C) 0 (D) –1

214. If the sides a, b, c of the triangle ABC are the roots of the equation x3 – px2 + qx – r = 0,
then its area is
1 1/ 2 1 1/ 2

(A) p 4pq  p3  8r
4
 (B)
2

p 2pq  p2  8r 
1/ 2

(C) p 4pq  p3  8r  (D) none of these

215. The two adjacent sides of a cyclic quadrilateral are 2 and 5 and the angle between them
is 60. If the third side is 3, the remaining fourth side is
(A) 2 (B) 3
(C) 4 (D) 5

216. If r1, r2, r3 are the ex–rodii and r is the in–radius of a triangle then the value of
r1 + r2 – r3 + r is
(A) 2R cos C (B) 2R sin C
(C) 4R cos C (D) 4R sin C
1 1 1
217. In a ABC, the value of   is
bc ca ab
1 1
(A) (B)
Rr 2Rr

FIITJEE Ltd., ICES House, 29-A, Kalu Sarai, Sarvapriya Vihar, New Delhi -110016, Ph 26515949, 26569493, Fax 26513942
FIITJEE Ltd. Material Provided by - Material Point Available on - Learnaf.com
M ast er A ssignm ent s P-57-P8-CBSE-MA-36

(C) Rr (D) 2Rr


218. If , , , are the distances of the vertices of a triangle from corresponding points of
contact with the in–circle, then
 2
(A) r 2  (B) r 2 
 

(C) r 2  (D) none of these
2     

 r  r 
219. If in a triangle ABC,  1  1   1  1   2, then the triangle is
 r2   r3 
(A) Isosceles (B) Equilateral
(C) Right angled (D) none of these

220. In a triangle ABC, the area of the in–circle to the area of the triangle itself is
(A)  : cot  A / 2  cot B / 2  cot  C / 2  (B)  : 2cot  A / 2  cot B / 2  cot  C / 2 
(C)  : tan  A / 2  tan B / 2  tan  C / 2  (D)  : 2 tan  A / 2  tan B / 2  tan  C / 2 

221. I is the in–centre of ABC and P1, P2 and P3 are respectively the radii of the circumcircles
of the triangles IBC, ICA and IAB. Then
r 2R R2r
(A) P1 P2 P3 = (B) P1 P2 P3 =
2 2
(C) P1 P2 P3 = R2r (D) P1 P2 P3 = 2R2r

222. Family of lines x(a + b) + y = 1 where a and b are roots of the equation x3  3x2 + x + 1 =
0 and [a + b] = 1 where [.] denotes the greatest integer, is such that it intercepts a triangle
of area A with the coordinate axes. Then Amax is
(A) 1 (B) 1/2
(C) 2 (D) 1/4

223. Consider two points A = (3, 0) and B = (0, 4). If a point P on the line 2x  3y  12 = 0 is
such that |PA  PB| is maximum, then P is
(A) (12, 12) (B) (6, 0)
(C) (0, 4) (D) none of these

224. If the family of lines x + 3y  6 = 0 ( is variable) intersect the lines x  2y + 3 = 0 and x


 y + 1 = 0 in P and Q, then locus of the middle point of PQ is
(A) 4x + 2y = 1 (B) x + y = 2
(C) 2x  2y + 4 = 0 (D) 4x + 3y = 4

225. Value of k for which the point (, sin1) lies inside the triagnle formed by x + y = k with
the coordinate axes belongs to
     
(A)  2  , 2   (B) 1  , 1  
 2 2  2 2
   
(C)    1,  1 (D) (1  sin1, 1 + sin1)
 2 2 

226. A line makes intercepts a and b with the coordinate axes. If the coordinate system is
rotated through 45 clockwise and the intercepts on the new system become A and B,
then which one is not true:

FIITJEE Ltd., ICES House, 29-A, Kalu Sarai, Sarvapriya Vihar, New Delhi -110016, Ph 26515949, 26569493, Fax 26513942
FIITJEE Ltd. Material Provided by - Material Point Available on - Learnaf.com

P-57-P8-CBSE-MA-37 Pinnacle Study Package

1 1 1 1
(A) 2
 2
 2
 2
(B) a2 + b2 = A2 + B2
a b A B
(C) aA = bB (D) none of these

227. If the curves ax2 + 4xy + 2y2 + x + y + 5 = 0 and ax2 + 6xy + 5y2 + 2x + 3y + 8 = 0
intersect at four concyclic points then the value of a is
(A) 4 (B)  4
(C) 6 (D)  6

228.   
Distance of the origin from the line 1  3 y  1  3 x  10 along the line y =  3x+k
is
5
(A) (B) 5 2  k
12
(C) 10 (D) 5

229. A line passes through (1, 0). The slope of the line, for which its intercept between y = x 
2 and y = x + 2 subtends a right angle at the origin is
2 3
(A)  (B) 
3 2
(C)  1 (D) none of these

230. Consider the line passing through    


3, 1 and 1, 3 . Then number of rational points
lying on the line is
(A) 1 (B) at most 2
(C)  (D) none of these

231. Let L1 = 0 and L2 = 0 be two intersecting straight lines. Then the number of points, whose
distance from L1 is 2 units and from L2 5 units is
(A) one (B) zero
(C) four (D) infinite

232. The maximum value of the sum of the lengths of intercepts made by any tangent to the
curve (asin2, 2a sin) with the axes is
(A) 2a (B) a/4
(C) a/2 (D) a

233. If x + y = a and x + 2y = 2a are the adjacent sides of a rhombus whose diagonals


intersect at (1, 2), then sum of all possible values of a is
(A) 5/3 (B) 10/3
(C) 5/3 (D) 10/3

234. A line x + y = 4 which intercepts the x-axis and the y-axis at A and B, is shifted by 1 units
towards the origin. The shifted line intercepts the x-axis and the y-axis at C and D. The
area of the trapezium ABCD is
(A) 4 2 units 
(B) 4 2  1 units 
(C) 4 
2  1 units (D)  8 2  1 units
235. If the lines 2x + 3y = 8, 5x  6y + 7 = 0 and px + qy = 1 are concurrent, then the line x +
2y  1 = 0 passes through
(A) (1, 2) (B) (p, 1)

FIITJEE Ltd., ICES House, 29-A, Kalu Sarai, Sarvapriya Vihar, New Delhi -110016, Ph 26515949, 26569493, Fax 26513942
FIITJEE Ltd. Material Provided by - Material Point Available on - Learnaf.com
M ast er A ssignm ent s P-57-P8-CBSE-MA-38

(C) (q, 1) (D) none of these

236. The lines joining the points of intersection of the curve 5x2 + 12xy  8y2  4y  8x + 12 = 0
and the line x  y = 2 to the origin, make angles with the positive x-axis which are
(A) equal (B) supplementary
(C) complimentary (D) none of these

237. Let a1, b1, c1 and a2, b2, c2 be the sides of two triangles such that the quadratic equations
a1x2 + b1x + c1 = 0 and a2x2 + b2x + c2 = 0 have a common root and b12  4a1c1 < 0. Then
the given triangles are
(A) concurrent (B) similar
(C) equilateral (D) right angled

238. The equations of sides of a triangle are x  3y = 0, 4x + 3y = 5 and 3x + y = 0. The line


3x  4y = 0 passes through its
(A) incentre (B) centroid
(C) circumcentre (D) orthocentre

239. If the area of quadrilateral formed by the pair of lines 4x2  4xy + y2 + 4x  2y = 0 and x2 +
4
2xy + y2  m2 = 0 is sq. units, then the values of m is
3
(A) 2 (B) 3
(C) 1 (D) 1/2

240. Let A, B, C be three points in a straight line. B lying between A and C. Consider all circles
passing through B and C. The points of contact of the tangents from A to these circle lie
on
(A) a straight line (B) a circle
(C) a Parabola (D) none of these

241. The equation of the circle which cuts the circle x2 + y2 = a2 orthogonally at the point
 a a 
 ,  such that slope of their common chord is –1, is
 2 2
(A) x 2  y2  2 2x  2 2y  a2  0 (B) x 2  y2  2 2x  2 2y  a2  0
(C) x 2  y 2  2x  2y  a2  0 (D) x 2  y2  2x  2y  a2  0

242. A line passing through the points A(1, –2) cuts the circle x2 + y2 – x – y = 0 at P and Q.
Then the maximum value of AP  AQ is
(A) 3 2 (B) 2
1
(B) 2 2 (D)
2

243. The equation of the circumcircle of an equililateral triangle is x2 + y2 + 2gx + 2fy + c = 0


and one vertex of the triangle is (1, 1). The equation of the incircle of the triangle is
2 2 2 2 2 2 2 2
(A) 4(x + y ) = g + b (B) 4(x + y ) + 8gx + 8fx = g + f
2 2 2 2
(C) 4(x + y ) – 8gx – 8fy = g + f (D) none of these

244. The circle x2 + y2 – 6x – 10y + c = 0 does not intersect or touch the coordinate axes and
has (1, 4) as its interior point. Exhaustive range of ‘c’ is
(A) (25, 29) (B) (9, 29)
(C) (9, 25) (D) (0, 9)

FIITJEE Ltd., ICES House, 29-A, Kalu Sarai, Sarvapriya Vihar, New Delhi -110016, Ph 26515949, 26569493, Fax 26513942
FIITJEE Ltd. Material Provided by - Material Point Available on - Learnaf.com

P-57-P8-CBSE-MA-39 Pinnacle Study Package

245. The circle x2 + y2 – 8x – 10y + 37 = 0 after being reflected about the line y = x is
shifted 2 2 units towards the origin, parallel to this line. Then the equation of the radical
axis of the given circle and the new circle is
(A) x + 3y – 14 = 0 (B) 3x + 4y – 21 = 0
(C) 6x + 2y – 7 = 0 (D) none of these

246. The circles having radii r1 and r2 intersect orthogonally. The length of their common chord
is
2r1r2 2r12r2
(A) (B)
r12  r22 r12  r22
r1r2 2r22r1
(B) (D)
r12  r22 r12  r22

247. Angle between tangents drawn to x2 + y2 – 2x – 4y + 1 = 0 at the points where it is cut by


the line y = 2x + c is /2. Then
(A) c = 5 (B) c = 2 5
(C) c = 10 (D) c = 2 10

248. The equation of the normal to the circle (x – 1)2 + (y – 2)2 = 4 which is at a maximum
distance from the point (–1, –1) is
(A) x + 2y = 5 (B) 2x + y = 4
(C) 3x + 2y = 7 (D) 2x + 3y = 8

249. If the circle (x – a)2 + y2 = 25 intersects the circle x2 + (y – b)2 = 16 in such a way that the
common chord is of maximum length, then value of a2 + b2 is
(A) 8 (B) 10
(C) 9 (D) 1
250. If the equation 3x2 + 3y2 + 6x + 2 = 0 represents a circle, then the value of  lies in
2  1 
(A) (– , 0)   ,   (B)  , 1   ,  
3  3 
2 
(C)  ,0    ,   (D) none of these
3 

251. Two circles which can be drawn to pass through (1, 0) and (3, 0) and to touch the
y–axis, intersect at an angle . Then cos  is equal to
(A) 1/2 (B) –1/2
(C) 1/4 (D) –1/4
252. 
If 1, 3  be one of the vertices of an equilateral triangle incircled in the circle x2 + y2 = 4,
then the other vertices are
(A)  2,0  , 1,  3    
(B) 1,  3 ,  2,0 

(C) 1, 3  , 1,  3  (D) none of these

253. Smallest circle passing through the points (a, 0) and (0, a)
(A) passes through the origin (B) touches the x–axis
(C) touches the y–axis (D) none of these
254. The equation of a circle touching the lines y  x at a distance 2 units from the origin is
2 2 2 2
(A) x + y + 4x = 0 (B) x + y – 4x + 2 = 0

FIITJEE Ltd., ICES House, 29-A, Kalu Sarai, Sarvapriya Vihar, New Delhi -110016, Ph 26515949, 26569493, Fax 26513942
FIITJEE Ltd. Material Provided by - Material Point Available on - Learnaf.com
M ast er A ssignm ent s P-57-P8-CBSE-MA-40

(C) x2 + y2 – 4x + 4 = 0 (D) none of these

255. A family of circles x2 + y2 – kx = 0 where k is a parameter such that none of the members
of the family is intersected at two distinct points by the line y + x = 2. Then the least value
of k  4 is
(A) 2 2 (B) 4
(C) 2 (D) 4 2
256. If the circle x2 + y2 + 2gx + 2fy + sin –1 = 0 passes through all the four quadrants, then
(A)   R (B)  is integral multiple of Q
(C)   R (D) none of these

257. If x2 + y2 = 1, then
2
(A) yy  2(y) + 1 = 0 (B) yy + (y) + 1 = 0
2 2
(C) yy  (y)  1 = 0 (D) yy + 2(y) + 1 = 0

258. If the line x  ay = 5 is a chord to the parabola y2 = 2x, then the circle with this chord as
diameter will always touch the line
(A) x + 5 = 0 (B) 2y + 5 = 0
(C) 2x + 5 = y (D) depend on a

259. If the focus of the parabola (y  k)2 = 4(x  h) always lies between the lines x + y = 1 and
x + y = 3, then
(A) 0 < h + k < 3 (B) 0 < h + k < 1
(C) 0 < h + k < 2 (D) 1 < h + k < 3

260. A line drawn through the focus (F) and parallel to tangent at P(x1, y1) on the parabola y2 =
4ax cuts the line y = y1 at Q. Then PQ is equal to
a
(A) a  x1 (B)  x1
2
(C) a + x1 (D) none of these
261. The line x  b + y = 0 cuts the parabola y2 = 4ax at P(at12, 2at1) and Q(at22, 2at2). If
b [2a, 4a] and   R, then t1t2 belongs to
(A) [4, 2] (B) [4, 3]
(C) [3, 2] (D) none of these

262. If the following informations is given about a parabola, then in which case the parabola is
not unique
(A) focus, equation of tangent at vertex
(B) focus, vertex
(C) equation of directrix, vertex
(D) equation of directrix, equation of tangent at vertex

263. If the focus of a parabola is (2, 3) and its latus rectum is 8, then locus of the vertex of the
parabola is
2 2
(A) (0, 3) (B) (x  2) + (y  3) = 4
2
(C) (y  3) = 8(x  2) (D) none of these
2
264. The point on the axis of the parabola 3y + 4y  6x + 8 = 0 from where 3 distinct normals
can be drawn is given by
 2 29 2  29
(A)  h,  where h  (B)  , h  where h 
 3 18 3  18

FIITJEE Ltd., ICES House, 29-A, Kalu Sarai, Sarvapriya Vihar, New Delhi -110016, Ph 26515949, 26569493, Fax 26513942
FIITJEE Ltd. Material Provided by - Material Point Available on - Learnaf.com

P-57-P8-CBSE-MA-41 Pinnacle Study Package

 1 29
(C)  h,  where h  (D) none of these
 3 18

265. If a circle with centre at (6, 5) meets the curve y2 = 4x orthogonally, then possible point of
intersection can be
(A) (9, 6) (B) (1, 2)
(C) (1, 2) (D) none of these

266. Given the parabola y2 = 8x and the point (2, 0). The area of the triangle formed by the
tangents from ‘P’ to the parabola and its chord of contact is
(A) 32 sq. units (B) 16 sq. units
(C) 8 sq. units (D) 4 sq. units

267. Consider a pair of straight lines one of which is a focal chord of the parabola y2 = 4ax and
the other is parallel to the axis of the parabola. If their point of intersection lies on the
parabola then their angle bisectors will be
(A) tangent to the parabola (B) normal to the parabola
(C) both (A) and (B) (D) none of these

268. If the tangents from (6, 8) to the parabola x 2 = 4y make angles 1 and 2 with the positive
direction of the x-axis, then |tan1  tan2| is equal to
(A) 2 (B) 3
(C) 3/4 (D) 4/5

269. The point on the axis of the parabola 3y2 + 6y  6x + 3 = 0 from which 3 distinct normals
can be drawn is given by
(A) (, 1),  > 2 (B) (1, ) ,  > 2
(C) (, 1),  > 1 (D) none of these

270. If from a point on the line (x  1) = 0, tangents are drawn to the parabola y2  2y  4x + 9
= 0 such that the pair of tangents and the chord of contact form a triangle, then the
minimum area of the triangle is
(A) 2 sq. units (B) 4 sq. units
(C) 6 sq. units (D) 8 sq. units

271. If the focus of the parabola y2 + 8 = 4x coincides with one of the foci of the ellipse
tx2 + 4y2  4tx = 0, then the value of t is
(A) 1/3 (B) 3
(C) 1/2 (D) none of these

272. A normal to the parabola y2 = 4ax with slope m touches the rectangular hyperbola
x2  y2 = a2 if
(A) m6 + 4m4  3m2 + 1 = 0 (B) m6  4m4 + 3m2  1 = 0
6 4 2
(C) m + 4m + 3m + 1 = 0 (D) m6  4m4  3m2 + 1 = 0

273. If the parabola y = ax2 + bx + c has vertex at (4, 2) and a  [1, 3], then difference
between the extreme values of abc is equal to
(A) 8600 (B) 144
(C) 3456 (D) none of these

274. The vertex V of the parabola y2 = 4ax and two points on the parabola together with a
point P form a square. The coordinates of P are
(A) (8a, 0) (B) (4a, 4a)
(C) (8a, 4a) (D) (4a, 0)

FIITJEE Ltd., ICES House, 29-A, Kalu Sarai, Sarvapriya Vihar, New Delhi -110016, Ph 26515949, 26569493, Fax 26513942
FIITJEE Ltd. Material Provided by - Material Point Available on - Learnaf.com
M ast er A ssignm ent s P-57-P8-CBSE-MA-42

x2 y2
275. If P(), Q( + /2) are two points on the ellipse   1 and  is the angle between
a2 b2
normals at P and Q, then
(A) 2 1  e2  e sin2 2  tan  (B) 2 1  e2  e sin2   tan2
(C) 1  e2  2e2 sin2 2  tan  (D) 2 1  e2  e2 sin2   tan 

276. The area between the latus rectum and tangents drawn at the end points of the latus-
x2 y 2
rectum of the ellipse 2  2  1 is
a b
b4 b2
(A) (B)
e2a2 e2a2
b4
(C) (D) none of these
ea2

277. If SK be the perpendicular from the focus S on the tangent at any point P on the
x2 y2
ellipse  1, then locus of K is
a2 b2
(A) a2x2 + b2y2 = (ax  by)2 (B) x2 + y2 = a2
(C) x2 + y2 = b2 (D) x2 + y2 = a2 + b2

278. A circle is a limiting case of an ellipse whose eccentricity


(A) tends to a (B) tends to 0
(C) tends to b (D) tends to (a + b)

x2 x2
279. If the curves  y 2  1 and 2  y 2  1 , for suitable value of ‘a’, cut on four concyclic
4 a
points, the equation of the circle passing through these four points is
(A) x2 + y2 = 2 (B) x2 + y2 = 1
2 2
(C) x + y = 4 (D) none of these

x2 y2
280. If eccentric angle of a point lying in the first quadrant on the ellipse  1 be  and 
a2 b2
line joining the centre to this point makes angel  with the x-axis, then    will be
maximum when  is equal to
b a
(A) tan1 (B) tan1
a b

(C) (D) none of these
4

x2 y2 2x x2 y 2 2x
281. If common tangents of the ellipses   and 2  2   0 subtends an
a2 b2 c b a c
angle  at the origin, then
 
(A)   (B)  
4 2
3
(C)   (D) none of these
2

FIITJEE Ltd., ICES House, 29-A, Kalu Sarai, Sarvapriya Vihar, New Delhi -110016, Ph 26515949, 26569493, Fax 26513942
FIITJEE Ltd. Material Provided by - Material Point Available on - Learnaf.com

P-57-P8-CBSE-MA-43 Pinnacle Study Package

x2 y2
282. If the ellipse   1 is inscribed in a square of side length 2a , then a is
a2  7 13  5a
equal to
6  13 
(A)
5

(B) ,  7   7, 


5 
 13 
 

(C) ,  7   , 7 
5 
(D) no such ‘a’ exists

x2 y2
283. The intercept made by the auxiliary circle of the ellipse  1 (a > b > 1) on any

a2 b2
tangent to the ellipse, subtends a right angle at its centre if
b b
(A) a  (B) a 
2 2
(C) a  2b (D) none of these

 3 3 x2 y 2
284. If normal at P  2,  meets the major axis of the ellipse   1 at Q, and S and
 2  16 9

S are foci of the given ellipse, then SQ : SQ is
8 7 4 7
(A) (B)
8 7 4 7
8 7 4 7
(C) (D)
8 7 4 7

285. If y = 2x + c neither cuts the circle (x  2)2 + (y  3)2 = 4 nor the ellipse x2 + 6y2 = 6, then
the range of c is
(A) [5, 5] (B) (, 5)  (5, )
(C) (4, 4) (D) none of these

286. If normals are drawn to the ellipse x2 + 2y2 = 2 from the point (2, 3), then the co-normal
point lie on the curve
(A) xy + 2x  3y = 0 (B) xy + 3x  4y = 0
(C) 2xy + 3x  4y = 0 (D) none of these
x2 y 2
287. A tangent at P meets the axes of the ellipse   1 in T and Q and CY is a
25 16
perpendicular on it from the centre. Then TQ.CY is equal to
(A) 144 (B) 9
(C) 41 (D) none of these
2 2
288. Tangent at any point P of the ellipse 9x + 16y  144 = 0 is drawn. Eccentric angle at P
1  1
is   sin1   . If N is the foot of the perpendicular from the centre O to this tangent
2 7
then PON is
 1   1 
(A) tan1   (B) tan1  
 12   24 

(C) (D) none of these
12

FIITJEE Ltd., ICES House, 29-A, Kalu Sarai, Sarvapriya Vihar, New Delhi -110016, Ph 26515949, 26569493, Fax 26513942
FIITJEE Ltd. Material Provided by - Material Point Available on - Learnaf.com
M ast er A ssignm ent s P-57-P8-CBSE-MA-44

289. The locus of the point of inter section of perpendicular chords passing through the foci of
x2 y 2
the ellipse 2  2  1 (a > b) is
a b
(A) x2 + y2 = a2 (B) x2 + y2 = a2 – b2
2 2 2
(C) x + y = b (D) x2 + y2 = a2 + b2

x2 y2
290. If normal at any point P to the ellipse  1 (a > b) meets the axes at A and B such
2

a b2
that PA : PB = 1 : 2, then the equation of the director circle of the ellipse is
a2 3a2
(A) x 2  y2  (B) x 2  y2 
2 2
2a2 a2
(C) x 2  y2  (D) x 2  y2 
3 3

x2
291. Given the ellipse  y 2  1 ; the point on the line x = 2, such that the tangents drawn
4
from the point to the ellipse are at 45°, is

(A) 2, 2  5   
(B) 2, 5

(C)  2, 3  (D)  2,  3 

292. If the sum of the slopes of the normals from a point P to the hyperbola xy = c2 is constant
k (k > 0), then the locus of point P is
(A) x2 = kc2 (B) x2 = kc
2 2
(C) x + kc = 0 (D) none of these

293. The line 2x + y = 0 passes through the centre of a rectangular hyperbola, one of whose
asymptotes is x  y = 1. The equation of the other asymptotes is
(A) 3x + 3y + 1 = 0 (B) 3x + 3y  1 = 0
(C) x  2y = 0 (D) none of these

294. If the portion of the asymptote between the centre and the tangent at the vertex of the
x2 y2
hyperbola 2
  1 in the third quadrant is cut by the line y + (x + a) = 0,  being
a b2
parameter, then
(A)   R+ (B)   R
(C)   (0, 1) (D) none of these

x2 y2
295. A normal to the hyperbola   1 has equal intercepts on the positive x and y axes.
4 1
x2 y 2 2 2
If this normal touches the ellipse   1, then a + b is equal to
a b
(A) 5 (B) 25
(C) 16 (D) 25/9

296. If values of m for which the line y = mx + 2 5 touches the hyperbola 16x2  9y2 = 144 are
2
the roots of the equation x  (a + b) x  4 = 0, then the value of (a + b) is equal to
(A) 2 (B) 4
(C) zero (D) none of these

FIITJEE Ltd., ICES House, 29-A, Kalu Sarai, Sarvapriya Vihar, New Delhi -110016, Ph 26515949, 26569493, Fax 26513942
FIITJEE Ltd. Material Provided by - Material Point Available on - Learnaf.com

P-57-P8-CBSE-MA-45 Pinnacle Study Package

297. If the chords of contact of tangents from two points (4, 2) and (2, 1) to the hyperbola
x2 y2
  1 are at right angles, then eccentricity of the hyperbola is
4 1
3
(A) 2 (B)
2
5
(C) (D) 3
2

298. If the curve xy = c (c > 0) and the circle x 2 + y2 = 1 touch at two points, then distance
between the points of contact, is
(A) 1 (B) 2
(C) 2 2 (D) none of these

299. The angle between the asymptotes of a hyperbola is 30°. The eccentricity of the
hyperbola may be
(A) 3  1 (B) 3  1
(C) 6  2 (D) 5  2

 1  1
300. If  a,  and  b,  be the extremities of a chord on the hyperbola xy = 1, then line
 a  b
perpendicular to this chord will be equally inclined to the coordinate axes for
(A) a2 = b2 (B) a2b2 = 1
2
(C) (a + b) = 1 (D) none of these
x2 y2
301. Let any double ordinate PNP of the hyperbola   1 be produced on both sides to
25 16
meet the asymptotes in Q and Q. Then PQ.PQ is equal to
(A) 25 (B) 16
(C) 41 (D) none of these
302. For a hyperbola whose centre is (1, 2), if asymptotes are parallel to the lines 2x + 3y = 0
and x + 2y = 1, then equation of the hyperbola passing through (2, 4) is
(A) (2x + 3y  5) (x + 2y  8) = 40 (B) (2x + 3y  8) (x + 2y  5) = 40
(C) (2x + 3y  8) (x + 2y  5) = 30 (D) none of these
2
303. If xy = 1 + sin  ( is real parameter) be a family of rectangular hyperbolas and  is the
area of the triangle formed by any tangent with coordinate axes, then
(A) 1.5    3 (B) 1    4
(C) 2    4 (D) 1    3

304. The focus of the rectangular hyperbola (x  y) (y  k) = p2 is


(A) (h  p, k  p) (B) (h  p, k + p)
(C) (h + p, k  p) (D) none of these

305. The chords (lx + my  1) = 0 (l, m being parameters) of the curve x2  3y2 + 3xy + 3x = 0,
subtending a right angle at the origin, are concurrent at the point
(A) (3/2, 0) (B) (3/2, 0)
(C) (1/2, 0) (D) none of these

306. If a rectangular hyperbola circum scribes a triangle, then it also passes through the
(A) incentre of the triangle (B) circumcentre of the triangle
(C) orthocentre of the triangle (D) none of these

FIITJEE Ltd., ICES House, 29-A, Kalu Sarai, Sarvapriya Vihar, New Delhi -110016, Ph 26515949, 26569493, Fax 26513942
FIITJEE Ltd. Material Provided by - Material Point Available on - Learnaf.com
M ast er A ssignm ent s P-57-P8-CBSE-MA-46

307. The foot of perpendicular from the focus on any tangent to a hyperbola lies on the
(A) conjugate hyperbola (B) auxiliary circle
(C) one of the asymptotes (D) director circle
308. If tangents OQ and OR, are drawn to variable circles having radius r and centre on the
rectangular hyperbola xy = 1, then locus of the circumcentre of triangle OQR is (O, being
origin)
(A) xy = 4 (B) xy = 1/4
(C) xy = 1 (D) none of these
x2 y2
309. If tangents PQ and PR are drawn from a variable point P to the hyperbola 1 2

a b2
(a > b), so that the fourth vertex S of the parallelogram PQSR lies on the circumcircle of
the triangle PQR, then locus of P is
(A) x2 + y2 = b2 (B) x2 + y2 = a2
2 2 2 2
(C) x + y = a  b (D) none of these

310. If foci of a hyperbola lie on y = x and one of the asymptote is y = 2x, then equation of the
hyperbola, given that it passes through (3, 4), is
5
(A) x2  y2  xy  5  0 (B) 2x2  2y2 + 5xy + 5 = 0
2
(C) 2x2 + 2y2  5xy + 10 = 0 (D) none of these

x2 y 2 x2 y2 1
311. If the foci of the ellipse  2  1 and the hyperbola   coincide, then
16 b 144 81 25
length of the latus rectum of the ellipse is
(A) 5/2 (B) 7/2
(C) 9/2 (D) none of these

312. If the variable curves y2 = 4(x2 + a) and y = bx2 + c (a, c are variable parameters and b is
constant) touch each other, then their point of contact lies on a
(A) straight line (B) circle
(C) parabola (D) hyperbola

x
313. The period of cos  x sin x  tan x
2
(A) is 2 (B) is 
(C) does not exist (D) none of these
2 2
314. The solution for |x  3| + |x  1| < 10 is
(A) ( 7, 3)  ( 3, 7 ) (B) ( 7, 7)
(C) ( 3, 3) (D) none of these

315. The functions f(x) = x and g(x) = x2 are identical for


(A) x  R (B) x  R+
(C) x  R  {0} (D) none of these

316. The period of the function f(x) = sin4x + cos4x is


(A)  (B) 2
(C) /2 (D) none of these

FIITJEE Ltd., ICES House, 29-A, Kalu Sarai, Sarvapriya Vihar, New Delhi -110016, Ph 26515949, 26569493, Fax 26513942
FIITJEE Ltd. Material Provided by - Material Point Available on - Learnaf.com

P-57-P8-CBSE-MA-47 Pinnacle Study Package

x 1
317. Domain and range of f(x)  is
x3  1
(A) R  {1}, (0, 1] (B) R, (0, 1]
(C) R  {1}, R (D) none of these

318. If f(x) = | x | + 1 and g(x) = x2  5, then range of g(f(x)) is


(A) [4, ) (B) (, )
(C) [4, 4] (D) none of these

319. The domain of log2log1/3 3x 2  5 is


 5 
(A)  , (B) ( 2, )
 3 
 
 5 
(C)  , 2 (D) none of these
 3 
 

320. If f(x  2) = x2  3x + 2, then f(x) is


(A) x2  x (B) x2 + x
(C) x2 + x + 1 (D) none of these

1
321. Domain of f(x)  is
2
x  x 1
(A) (, ) (B) (, )  {1}
(C) [0, ) (D) none of these

322. If sinbx + cosax is a periodic function, then


(A) a, b are irrational (B) a is rational, b is irrational
(C) b is rational, a is irrational (D) a, b are rational

 x2 x  0
323. If f(x)   and g(x) = x2 and h(x) = 2x + 4, then number of solutions of g(f(x))
3x  2 x  0
= h(x) is
(A) 1 (B) 2
(C) 3 (D) 4

324. The period of sinx + {x} (where { } is fractional part of x) is


(A)  (B) 1
(C) 2 (D) none of these

325. The number of solutions of 1 + x cosx = sinx is


(A) 1 (B) 2
(C) infinite (D) none of these

326. If f(x) is an even function, then f(x) and f(x) are


(A) even, even (B) odd, even
(C) even, odd (D) odd, odd
2
327. If f(x) = |ln|x|| and g(x) = |ln|x|| and h(x) = x  4, then total number of solutions of h(x) =
f(x) and h(x) = g(x) is
(A) 2 (B) 4
(C) 6 (D) 8

FIITJEE Ltd., ICES House, 29-A, Kalu Sarai, Sarvapriya Vihar, New Delhi -110016, Ph 26515949, 26569493, Fax 26513942
FIITJEE Ltd. Material Provided by - Material Point Available on - Learnaf.com
M ast er A ssignm ent s P-57-P8-CBSE-MA-48

328. The set of values of a for which function log2log3 log1/4log5(a  6) is defined is
(A) (1, ) (B) (, 6)
(C)(6, 11) (D) (6, )

tan2 [x]
329. lim , where [ ] represents greatest integer function, is
x 0 [x]2
(A) 1 (B) 1
(C) 0 (D) does not exist

330. lim {x} , where { } represent fractional part of x, is


x 1/ 2
(A) 0 (B) 1
(C) 1/2 (D) does not exist

1/{x}
331. 
lim log
x 1
2
2x  , where { } represent fractional part of x, is

(A) 2 (B) e
log e
(C) e 2
(D) none of these

4 2  (cos x  sin x)5


332. lim is
x  / 4 1  sin2x
(A) 5 (B) 5 2
(C) 2 (D) none of these

333. If f(x) = 1 + |x  1| and g(x) = 3  |x + 1|, then g(f(x))is not continuous at the points
(A) {1, 1} (B) {1}
(C) {1} (D) none of these

| x 2  1|
334. lim is
x 1
x 4  4x 2  7
(A) 1 (B) 2
(C) 0 (D) none of these

x 1
335. If the function f(x) satisfies the relation f(x  y)  y f(x)  f(y) with f(1) = 2, then
(x  1)
lim f (x) is
x 1
(A) 2 (B) 2
(C) 0 (D) none of these
4 2 2
336. The value of y(1) if x + 2x y  5x + y  2 = 0, where y(1) = 1, is equal to
3 2
(A) (B)
5 5
2 3
(C) (D)
5 5
337. If f : R  R is a function which is defined by f(x) = max{x5, x3}, the set of all the points
where f(x) is not differentiable is
(A) {1, 1} (B) {1}
(C) {1} (D) {1, 0, 1}

FIITJEE Ltd., ICES House, 29-A, Kalu Sarai, Sarvapriya Vihar, New Delhi -110016, Ph 26515949, 26569493, Fax 26513942
FIITJEE Ltd. Material Provided by - Material Point Available on - Learnaf.com

P-57-P8-CBSE-MA-49 Pinnacle Study Package

338. If f(x) = |x2  2| and g(x) = f(f(x)), then lim g(x) is


x 2
(A) 2 (B) 2
(C) 0 (D) none of these

339. If f(x) = 52x+1 and g(x) = 5x + 3x log5, then solution for f(x) > g(x) is
log(1/ 2) log(3 / 5)
(A) x  (B) x 
log5 log(5)
log( 1/ 2) log(3 / 5)
(C) x (D) none of these
log(5) log5

340. lim ([sin x]  x) is


x 
(A)  (B) 1 + 
(C)   1 (d) does not exist

ax a 1
341. If f(x)   2 , then the values of a, b, for which f(x) is differentiable, are
ax  bx  2 a  1
3 1
(A) a  , b  (B) a = 2, b = 2
4 4
3 1 3
(C) a  , b  (D) a  , b  2
2 4 4

342. Maximum slope of the curve y = 3x3 + 3x2 + 2x  27 is


(A) 2 (B) 2
(C) 3 (D) 3

343. Minimum value of the function f(x) = sec2(x ln[x] + 2x2  e[x]), given that x  [1, ), is
(A) 2 (B) 3/2
(C) 0 (D) none of these

344. Let f(x) = x3 + kx2 + 5x + 4sin2x be an increasing function on x  R. Then domain of k is



(A)  2, 2  (B)  , 3  
(C)  3,   (D)   3, 3 
x
345. For x  0, F(X) =  f  t  dt and f(x) is a continuous functions. If, for some c > 0, cF(x)  f(x),
0
then f(x) is equal to
(A) ex (B) x
(C) 0 (D) none of these
2 2
346. The interval of increase and decrease of the function f(x) = x + 5x + 4 + |x  1|, x  R, is
(A) increase [1, ) : decrease (, 1]
(B) increase [1, 1] : decrease (, 1]
(C) increase [1, ) : decrease [1, 1]
(D) none of these

FIITJEE Ltd., ICES House, 29-A, Kalu Sarai, Sarvapriya Vihar, New Delhi -110016, Ph 26515949, 26569493, Fax 26513942
FIITJEE Ltd. Material Provided by - Material Point Available on - Learnaf.com
M ast er A ssignm ent s P-57-P8-CBSE-MA-50

2 2 
347. The tangent to the curve f() = sin   cos   2 at x  is
4
 
(A) y  (B) x 
2 2
 
(C) x   (D) y  
2 2

 1 x 
348. The equation of the tangent to the curve f(x)  ln   at x =  1 is
 1 x 
(A) y =  1 (B) x  y = 2
(C) x =  1 (D) none of these

x2  1
349. If f(x) = for x  (1, 1), then
1  x sin x
(A) f(x) has exactly one point of maximum
(B) f(x) has exactly one point of maximum
 
(C) f(x) is increasing in  0, 
 2
(D) none of these

350. The function f(x) = tan1x  ln(1 + x2) is increasing for


1 1
(A) x  (B) x 
2 2
 1 1  1 
(C) x    ,  (D) x   2 ,  
 2 2  

351. The equation of the tangent to the curve f(x) = [x] (where [x] represent greatest integer
5
function) at x  is
2
(A) x = 5 (B) y = 5
(C) y = 2 (D) x = 2
x3  3x 2  x  3
352. Extreme values of the function f(x)  exist at
x2  1
(A) x = 1, 3 (B) x = 1
(C) x = 3 (D) none of these

353. All the tangents to the curve y = x3 + x + 1 make angles with the x-axis which are always
(A) acute (B) obtuse
(C) right angle (D) none of these
2 3 5 2
354. Let the given function be f(x) = x + sin x  2x cos x sinx. Then tangent to the curve at its
points of maxima and minima is
(A) parallel to the x-axis (B) parallel to the y-axis
(C) equally inclined to both the axis (D) none of these

dx
355.  x  9  x2  4
2
=

FIITJEE Ltd., ICES House, 29-A, Kalu Sarai, Sarvapriya Vihar, New Delhi -110016, Ph 26515949, 26569493, Fax 26513942
FIITJEE Ltd. Material Provided by - Material Point Available on - Learnaf.com

P-57-P8-CBSE-MA-51 Pinnacle Study Package

  2 

1 1  
x  x2  4  
(A)  
5 2  x 2  9  x 2  4  log   9/2

 
 x  x2  4
 
 
  2 
 x  x  4 
2
1 1 
(B) 
5 2  2
x 9  x  4   log 
2
9/2

  
 x x 42 
  
  2 
 x  x  4 
2
1 1 
(C) 
5 2  2
x 9  x  4   log 
2
9/2


  
 x x 4
2 
  
(D) none of these

e x  x3  x  2   8 
  dx =
356.   x  2 2
 8   8 
(A) e x  x2  4x  8  (B) e x  x2  2x  4 
 x  2   x  2 
 8 
(C) e x  x2  4x  8  (D) none of these
 x  2 

x 3  6x 2  10x  2
357.  x 2  5x  6
dx =

x 1 x3
(A) x2 + x + log 2
(B) x2  x + log
 x  2  x  2 2
2 2
 x  2  x  2
(C) x2 + x + log (D) x2  x + log
x3 x3

cos 4x  1
358.  cot x  tan x dx =
1 1
(A) cos 4x (B) sin 4x
8 8
1 1
(C)  cos 4x (D)  sin 4x
8 8

359. Let f(x) be a polynomial of degree three such that f(0) = 1, f(1) = 2. If 0 is a critical point
f  x
of f(x) which doesn’t have local extremum at x = 0 then
x2  7
dx 

1 2 3/2 1/ 2
(A)
3

x 7 
 7 x2  7  
 log x  x2  7  c

1 2 3/2 1/ 2
(B)
3

x 7 
 7 x2  7  
 log x  x 2  7  c

1 2 3/2 1/ 2
(C)
3

x 7 
 7 x2  7  
 log x  x2  7  c

(D) none of these

FIITJEE Ltd., ICES House, 29-A, Kalu Sarai, Sarvapriya Vihar, New Delhi -110016, Ph 26515949, 26569493, Fax 26513942
FIITJEE Ltd. Material Provided by - Material Point Available on - Learnaf.com
M ast er A ssignm ent s P-57-P8-CBSE-MA-52

4
360.  sec xdx =

tan3 x tan3 x
(A) tan x  c (B) tan x  c
3 3
tan2 x
(C) tan x  c (D) none of these
3
x
361.  1  x3
dx =

(A) log x3 / 2  1  x3  c (B) log x3 / 2  1  x3  c

(C) log x3  1  x3  c (D) none of these

dx
362.  x 2

 1 x2  4 
=

tan1 x 1 x tan1 x 1 x
(A)  tan1  c (B)  tan1  c
3 6 2 3 6 2
tan1 x 1 x
(C)  tan1  c (D) none of these
6 6 2
dx
363. If x 2
 fog  x   c then
 x 1
3 2x  1 2 2x  1
(A) f(x) = tan1 x, g  x   (B) f(x) = tan1 x , g(x) =
2 3 3 3
2 2x 1
(C) f(x) = tan1 x, g  x   (D) none of these
3 3

x 2
2 
364.  x 2

1 x  4 2

dx =

1  2x  1  3x 
(A) tan1  2 c (B) tan1   2 + c
2  x 2 3  x 2
1  2x 
(C) tan1  2 c (D) none of these
2  x  2
 x  1
365.  x 1  xe 2
dx =
  x

xe x xe x 1
(A) log  
 1  xe x  c (B) log  c
1  xe x
1  xe x
1  xe 
x

1  xex
(C) log
xe x  
 1  xe x  c (D) none of these

366. The function y = f(x), satisfying f(x) = x3/2, f(4) = 2 and f(0) = 0, is
(A) 3x  4 x (B) 3x  4 x
(C) 3x  4 x (D) none of these

FIITJEE Ltd., ICES House, 29-A, Kalu Sarai, Sarvapriya Vihar, New Delhi -110016, Ph 26515949, 26569493, Fax 26513942
FIITJEE Ltd. Material Provided by - Material Point Available on - Learnaf.com

P-57-P8-CBSE-MA-53 Pinnacle Study Package

cos x
367.  cos  x    dx =
(A) (x + ) cos + sin logsec(x + ) + c
(B) (x  ) cos + sin logsec(x  ) + c
(C) (x + ) cos  sin logsec(x  ) + c
(D) none of these
2
2x.2x
368.  1  4x2
dx =

(A) log2 e sin1 2 x  c2


(B) log2 2sin1 2x  c 2

(C) log2 e sin1 2x  c   (D) none of these

dx
369.  1  4 cos 2
x
=

1  tan x  1  tan x 
(A) tan1  c (B) tan1  c
2 5  5  2  5 
1  tan x  1  tan x 
(C) tan1  c (D) tan1  c
5  5  2 5  5 

x tan1 x2   dx =
370.  1 x4
1 1 2
(A) tan1 x2  c
2
(B)
4
tan1 x2  c

1 2

(C) tan1 x  c
4
 (D) none of these

x 1
371. a dx =

a x 1
(A) c (B) ax+1 loga
loga
ax loga
(B) (D) none of these
a
dx
372.  8
=
1  x 
1 1
(A) 7 7 
x 1  c (B) 7 7 
x 1  c
21 1  x   
21 1  x 
1
(C) 7 7 
x 1  c (D) none of these

21 1  x 
1.5
373.  x dx ( where {} is frictional part of x) is equal to
0
(A) 0.625 (B) 0.6

FIITJEE Ltd., ICES House, 29-A, Kalu Sarai, Sarvapriya Vihar, New Delhi -110016, Ph 26515949, 26569493, Fax 26513942
FIITJEE Ltd. Material Provided by - Material Point Available on - Learnaf.com
M ast er A ssignm ent s P-57-P8-CBSE-MA-54

(C) 0.725 (D) 0.7

2
2
374.  1 x
2
dx is equal to

(A) 0 (B) 2
(C) 4 (D) 8
2
1
375. If for non zero x, a f(x) + b f(1/x) =  5 where a  b then
x  f  x  dx
1
is equal to

 1 7   1 7 
(A) 2  alog2  5a  b 
2
(B) 2 alog2  5a  b 
2
a b  2  a b  2 
1  7 
(C) 2 2 
alog2  5a  b  (D) none of these
a b  2 
3
x 2 sin x
376. 
3
1  x6
dx is equal to

(A) 4 (B) – 4
(C) 0 (D) none of these

x sin2x sin    / 2  cos x 
377. 
0
2x  
dx is equal to

2 
(A) (B)
8 8
8 8
(C) (D)
 2
1
log 1  x 
378. 
0
1 x2
dx is equal to

 
(A) log2 (B) log4
4 8
 
(C) log2 (D) log2
8 4
4
2x 1
379. The value of 3
0
dx is equal to

6  4  66
(A)  13   (B)
log3  log3  log3
6  5 
(C)  13   (D) none of these
log3  log3 

2n
1 r
380. lim
n n
2
r 1 n  r
2
is equal to

(A) 1  5 (B) 1  2
(C) 1  2 (D) 1  5

FIITJEE Ltd., ICES House, 29-A, Kalu Sarai, Sarvapriya Vihar, New Delhi -110016, Ph 26515949, 26569493, Fax 26513942
FIITJEE Ltd. Material Provided by - Material Point Available on - Learnaf.com

P-57-P8-CBSE-MA-55 Pinnacle Study Package

/2
sinnx
381. If n = 2m + 1, m  N  {0}, then 
0
sin x
dx is equal to


(A)  (B)
2

(C) (D) none of these
4

 ax
382. e
0
cosbx dx is equal to

a a
(A) 2 2
(B)
a b a  b2
2

b b
(C) (D)
a2  b2 a2  b2

 
1 1 1 1
383. lim      is
n  2 2 2 2 2 2 
 n n 1 n 2 n   n  1 

 
(A) (B)
4 2
(C) does not exist (D) none of these

y x2
2 sin t dy
384. If  cos t dt  
0 0
t
dt then
dx
is equal to

2sin x 2sin x2
(A) (B)
x cos y x cos y 2
2 sin x 2 2 sin x
(C) (D)
x cos y x cos y 2

385. The function f(x) is defined as follows:


f(x) = x2, –1  x < 1
= x,1x<2
= 2 , 2  x  4.
4
Then  f  x  dx is
1

10 10 2
(A) (B)
3 3
(C)10 2 (D) none of these

100
386.  x dx (where {x} is fractional part of x) is equal to
10
(A) 90 (B) 40
(C) 45 (D) 95

FIITJEE Ltd., ICES House, 29-A, Kalu Sarai, Sarvapriya Vihar, New Delhi -110016, Ph 26515949, 26569493, Fax 26513942
FIITJEE Ltd. Material Provided by - Material Point Available on - Learnaf.com
M ast er A ssignm ent s P-57-P8-CBSE-MA-56

/3
1
387.  1
/6
tan x
dx is equal to

 
(A) (B)
6 3
 
(C) (D)
2 12

3
388. If f  x   x 2  x  x  2 then  f  x  dx is equal to
0
22 20
(A) (B)
3 3
22
(C) (D) none of these
2
/2
389.  x
0

tanx  cot x dx is equal to

 2
(A) (B)
2 2 2 2
 2
2
(C) (D)
2 2
30
3
390.  
0
x  dx (where [.] is integral part of x) is equal to

(A) 50 (B) 52
(C) 54 (D) 56

391. The area bounded by the curves y = f(x), the xaxis and the ordinates x = 1 and x = b is
(b  1)sin (3b + 4). Then f(x) is
(A) (x  1) cos (3x + 4) (B) sin (3x + 4)
(C) sin (3x + 4) + 3(x  1) cos (3x + 4) (D) none of these

392. Let g(x) be a function defined on [ 1, 1]. If the area of the equilateral triangle with two of
3
its vertices at (0, 0) and (x, g(x)) is , then the function g(x) is
4
(A)  1  x 2 (B) 1  x2
(B)  1  x 2 (D) 1  x2

393. Area of the region bounded by the xaxis, the curve C: y = tan x and the tangent to C at x

= , is
4
1 1 1
(A) log 2  (B) ln2 
4 2 4
1 1
(C) ln2  (D) none of these
2 4

FIITJEE Ltd., ICES House, 29-A, Kalu Sarai, Sarvapriya Vihar, New Delhi -110016, Ph 26515949, 26569493, Fax 26513942
FIITJEE Ltd. Material Provided by - Material Point Available on - Learnaf.com

P-57-P8-CBSE-MA-57 Pinnacle Study Package

2
394. Area of the region bounded by y = x2 and y = is
1  x2
 1 2
(A)  (B)  
2 3 3
1
(C)   (D) none of these
3
st
395. The area bounded by the curves y = x , 2y + 3 = x and the xaxis, in the 1 quadrant, is
27
(A) 9 (B)
4
(C) 36 (D) 18
396. The area bounded by the curves y = x  1 and y =  x  1 is
(A) 1 (B) 2
(C) 2 2 (D) 4

397. If An is the area bounded by the curve y = (tan x)n and the lines x = 0, y = 0 and x = ,
4
then, for n > 2, An + An  2 is equal to
1 1
(A) (B)
n n 1
1 n
(C) (D)
n1 n2  1
9
398. If the area of the region bounded by the curve y = x  x2 and the line y = mx equals ,
2
then m can have the value
(A)  4 (B)  2
(C) 2 (D) 4
d  d2 y 
399. If y2 = P(x), a polynomial of dgree 3, then 2  y 2 2  is equal to
dx  dx 
(A) P(x) + P(x) (B) P(x) P(x)
(C) P(x) P(x) (D) a constant

2
 x2  4   dy 
400. If x = sec   cos , y = secn   cosn , then  2    is equal to
y  4   dx 

2
(A) n + 1 (B) n2 + n + 1
(C) n2 (D) none of these

401. The primitive integral of the differential equation (x2 + y2)dy = xydx is y = y(x). If y(1) = 1,
and y(x0) = e, then x0 is equal to

(A) 2 e2  1  (B) 
2 e2  1 
e2  1
(C) 3e (D)
2

402. The primitive integral of the differential equation ydx + y2dy = xdy; x  R, y > 0, is y = y(x).
If y(1) = 1, then y(3) is equal to
(A) 3 (B) 2
(C) 1 (D) 5

FIITJEE Ltd., ICES House, 29-A, Kalu Sarai, Sarvapriya Vihar, New Delhi -110016, Ph 26515949, 26569493, Fax 26513942
FIITJEE Ltd. Material Provided by - Material Point Available on - Learnaf.com
M ast er A ssignm ent s P-57-P8-CBSE-MA-58

ANSWERS

1. A 2. D 3. A 4. B
5. B 6. D 7. C 8. A
9. D 10. B 11. C 12. D
13. B 14. B 15. D 16. C
17. B 18. B 19. C 20. A
21. B 22. A 23. C 24. A
25. A 26. A 27. C 28. D
29. A 30. D 31. A 32. A
33. D 34. C 35. B 36. A
37. C 38. A 39. A 40. C
41. A 42. B 43. C 44. D
45. A 46. B 47. A 48. C
49. C 50. C 51. B 52. A
53. D 54. D 55. C 56. B
57. C 58. B 59. B 60. A
61. A 62. A 63. C 64. B
65. A 66. B 67. C 68. A
69. A 70. C 71. C 72. B
73. A 74. B 75. C 76. B
77. B 78. C 79. C 80. A
81. B 82. D 83. C 84. A
85. B 86. C 87. B 88. A
89. A 90. A 91. A 92. C
93. C 94. C 95. C 96. D
97. D 98. C 99. D 100. B
101. A 102. C 103. C 104. B
105. C 106. A 107. A 108. B
109. B 110. D 111. A 112. A
113. A 114. B 115. A 116. B
117. B 118. A 119. A 120. D
121. A 122. A 123. C 124. C
125. A 126. C 127. D 128. A
129. B 130. A 131. D 132. A
133. B 134. A 135. A 136. B
137. B 138. B 139. B 140. A
141. C 142. B 143. A 144. B
145. B 146. D 147. A 148. C
149. C 150. A 151. C 152. B
153. B 154. B 155. B 156. C
157. B 158. C 159. C 160. B
161. B 162. D 163. A 164. B
165. C 166. A 167. B 168. D
169. D 170. C 171. C 172. A
173. A 174. B 175. C 176. C
177. B 178. A 179. B 180. A
181. D 182. B 183. A 184. C
185. A 186. B 187. C 188. C
189. A 190. B 191. D 192. B
193. B 194. A 195. B 196. B
197. B 198. B 199. A 200. C
201. C 202. B 203. A 204. C
205. A 206. D 207. C 208. A
209. B 210. B 211. A 212. C

FIITJEE Ltd., ICES House, 29-A, Kalu Sarai, Sarvapriya Vihar, New Delhi -110016, Ph 26515949, 26569493, Fax 26513942
FIITJEE Ltd. Material Provided by - Material Point Available on - Learnaf.com

P-57-P8-CBSE-MA-59 Pinnacle Study Package

213. C 214. A 215. A 216. C


217. B 218. A 219. C 220. A
221. D 222. B 223. A 224. D
225. B 226. A 227. B 228. D
229. D 230. D 231. C 232. A
233. B 234. C 235. D 236. B
237. B 238. D 239. C 240. B
241. D 242. B 243. A 244. A
245. A 246. A 247. C 248. D
249. C 250. C 251. A 252. A
253. A 254. B 255. D 256. C
257. B 258. A 259. C 260. D
261. A 262. D 263. B 264. A
265. A 266. B 267. C 268. A
269. C 270. B 271. B 272. C
273. C 274. A 275. D 276. C
277. B 278. B 279. B 280. B
281. B 282. D 283. D 284. A
285. B 286. B 287. B 288. B
289. B 290. B 291. A 292. A
293. A 294. D 295. D 296. C
297. B 298. B 299. C 300. B
301. B 302. B 303. C 304. A
305. B 306. C 307. B 308. B
309. C 310. C 311. B 312. D
313. C 314. B 315. B 316. C
317. A 318. A 319. C 320. B
321. A 322. D 323. C 324. C
325. C 326. D 327. C 328. C
329. D 330. C 331. C 332. B
333. B 334. C 335. D 336. D
337. A 338. C 339. B 340. D
341. B 342. C 343. D 344. D
345. C 346. A 347. D 348. C
349. B 350. A 351. C 352. C
353. A 354. A 355. A 356. C
357. B 358. A 359. A 360. B
361. A 362. B 363. C 364. B
365. B 366. B 367. A 368. A
369. A 370. B 371. A 372. B
373. A 374. C 375. C 376. C
377. D 378. C 379. D 380. D
381. B 382. B 383. B 384. D
385. B 386. C 387. D 388. A
389. B 390. C 391. C 392. C
393. B 394. B 395. A 396. B
397. B 398. B 399. C 400. C
401. C 402. A 403. C 404. B
405. C

FIITJEE Ltd., ICES House, 29-A, Kalu Sarai, Sarvapriya Vihar, New Delhi -110016, Ph 26515949, 26569493, Fax 26513942
FIITJEE Ltd. Material Provided by - Material Point Available on - Learnaf.com
M ast er A ssignm ent s P-57-P8-CBSE-MA-60

MULTI-CHOICE
(More than one correct answer)
Indicate all the correct answers by making appropriate choices from A, B, C and D
2 n 2n
1. If (1 + x + x ) = a0 + a1x + a2x + …. + a2nx , then the value of a0+ a3 + a5 + … is
(A) a1 + a4 + a7 + … (B) a2 + a5 + a8 + ….
(C) 3n1 (D) none of these

2. The largest coefficient in the expansion of (4 + 3x )25 is


11 11
25 25 4 3
(A) C11 . 3   (B) 25C4 . 425  
3 4
(C) 25C4 . 414 311 (D) none of these

A A A 
3. If 1  sin A  sin  cos , then  may lie in
2 2 2 4
(A) the first quadrant (B) the second quadrant
(C) the third quadrant (D) the fourth quadrant

1
4. If sinx + cosx = y , x  [0, ], then
y
 1
(A) x = (B) y =
4 2
3
(C) y = 1 (D) x =
4

5. If sin( cos) = cos( sin), then sin2 is equal to


3 3
(A) (B) 
4 4
1 1
(C) (D) 
4 4

6. A general solution of the equation tan2 + cos2  1 = 0 is



(A)  = n (B)  = 2n +
4
 
(C)  = n + (D)  = 2n 
4 4

7. Let ax + by + c = 0 represent a family of straight lines, where a, b, c are the 1st, 3rd and
7th terms respectively of an increasing A.P. Each member of the family passes through a
fixed point lying on the curve
(A) x2 + y2 = 13 (B) x2 + y2 = 5
9
(C) y2 = x (D) 3x + 4y = 9
2

8. Equations of the sides AB, BC, CA of the triangle ABC respectively are x = 2, y = 5,
5x + 2y = 10. The orthocentre of the triangle lies on the line
(A) x  y =0 (B) 3x  y = 1
(C) 4x + y = 13 (D) x  2y = 1

FIITJEE Ltd., ICES House, 29-A, Kalu Sarai, Sarvapriya Vihar, New Delhi -110016, Ph 26515949, 26569493, Fax 26513942
FIITJEE Ltd. Material Provided by - Material Point Available on - Learnaf.com

P-57-P8-CBSE-MA-61 Pinnacle Study Package

9. Tangents drawn from the origin to the circle x2 + y2 + 2gx + 2fy + f 2 = 0 are perpendicular
if
(A) g = f (B) g =  f
(C) g = 2f (D) 2g = f

10. A circle makes intercepts of length 2a and 3b on the coordinates axes. The equation of
the tangent from the origin to the circle is
(A) ax + by = 0 (B) x = y
(C) ax  by = 0 (D) none of these
2 2 2 2
11. The circles x + y  6x  4y  12 = 0 and x + y  4x  6y  12 = 0
(A) are orthogonal (B) have 2 common tangents
(C) have radical axis x = y (D) none of these
2 2
12. Tangents are drawn from a point P on the line y = 4 to the circle x + y = 25. If the
tangents are perpendicular to each other, then coordinates of P are
(A)  17, 4 (B)  34, 4 
(C)   34, 4  (D)   17, 4 

13. A square is inscribed in the circle x2 + y2  10x  6y + 30 = 0. One side of the square is
parallel to the straight line y = x + 3. Then one of the vertices of the square is
(A) (3, 3) (B) (7, 3)
(C) 5, 5) (D) (9, 5)

14. If f(x + y) + f(x  y) = 2f(x) f(y)  x, y  R, and f(0) = k, then


(A) f(x) is even if k = 1 (B) f(x) is odd if k = 0
(C) f(x) is odd for all k (D) f(x) is neither even nor odd for any k

15. If a > 0, b > 0, c> 0 and a + b + c = abc, then at least one of a, b or c exceeds
3 17
(A) (B)
2 10
13
(C) 2 (D)
10

16. If  (x) = f(x) + f(1 x), and f(x) < 0 for 0 < x < 1, then
 1  1
(A) (x) increases in 0,  (B) (x) decreases in 0, 2 
 2  
1  1 
(C) (x) increases in  , 1 (D) (x) decreases in  2 , 1
2   


 x 2  4x 3  x  0

 
17. Let f(x) =  sin x 0x . Then
 2
 
 cos x  1 2
x

(A) f(x) has global maximum at x =  2 (B) f(x) is discontinuous at x = 0


 
(C) f(x) is non-differentiable at x = (D) f(x) has global minimum at x =
2 2

FIITJEE Ltd., ICES House, 29-A, Kalu Sarai, Sarvapriya Vihar, New Delhi -110016, Ph 26515949, 26569493, Fax 26513942
FIITJEE Ltd. Material Provided by - Material Point Available on - Learnaf.com
M ast er A ssignm ent s P-57-P8-CBSE-MA-62

18. If [.] denotes the greatest integer function, then the function
  
f(x) = cos   cos   x  1  is continuous at
x 2 
(A) x = 0 (B) x = 1
(C) x = 2 (D) none of these

 x2  2x
 x  0, 2
 x2  2x

19. If f(x) 1 x0 , then f(x) is discontinuous at
 1 x2



(A) x = 1 (B) x = 0
(C) x = 1 (D) x = 2

20. If 1 < a < 2 and the equation 8x3  ax2 + bx  1 = 0 has real roots in G.P., then the
ordered pair (1, 2) is
(A) (2, 2) (B) (24, 29)
(C) (10, 8) (D) none of these


21. If f(x) = (sin2x  1)n (2 + cos2x), then x = is a points of
2
(A) local maximum, if n is odd (B) local minimum, if n is odd
(C) local maximum, if n is even (D) local minimum if n is even

22. If f(x) = tan1(sinx + cosx), then f(x) is increasing in


     
(A)   ,  (B)  4, 4 
 2 4  
 5 3   7  
(C)  ,  (D)  2, 4 
 4 2   

x y 4
23. If   1 is a tangent to the curve x = 4t, y = , then
a b t
(A) a > 0, b > 0 (B) a > 0, b < 0
(C) a < 0, b > 0 (D) a < 0, b< 0

24. If the function f(x) = (x2  1)n+1 (x2 + x + 1) has local extremum at x = 1, then n can have
value
(A) 2 (B) 3
(C) 4 (D) 5
2
25. If the line (3  a)x + ay + a  1 = 0 is a normal to the curve xy = 1, then a belongs to
(A) (3, ) (B) (, 0)
(C) (0, 3) (D) none of these

e
26. If  ln x  dx  a  eh ,
1
then

(A) a = 4 (B) a = 1
(C) b = 1 (D) b = 0

FIITJEE Ltd., ICES House, 29-A, Kalu Sarai, Sarvapriya Vihar, New Delhi -110016, Ph 26515949, 26569493, Fax 26513942
FIITJEE Ltd. Material Provided by - Material Point Available on - Learnaf.com

P-57-P8-CBSE-MA-63 Pinnacle Study Package

27. A circle, with its centre at the focus of the parabola y2 = 4ax and touching its directrix,
intersects the parabola at the point
(A) (a, 2a) (B) (a,  2a)
a  a 
(C)  , 2a  (D)  ,  2a 
 2   2 

28. If three distinct normals are drawn to the parabola y2 = x from the point (c, 0), then
1 1
(A) c  (B) c 
6 2
2
(C) c  (D) c  1
3

29. If tangents to the parabola y2 = 4ax are normals to the parabola x2 = 4y, then
(A) a   2 2 (B) a = 1
(C) a =  1 (D) a  2 2

30. If the line 3x + 4y + 6 = 0 is the axis of the parabola with vertex ( 2, 0), and latus rectum
4, then its equation is
(A) (3x + 4y)2 = 44x  108y + 124 (B) (3x + 4y)2 = 108x – 44y + 252
2
(C) (3x + 4y) = 116x + 12y  196 (D) (3x + 4y)2 = 36x + 48y + 108

31. If the tangents to the parabola y2 = 4x drawn from the point (x2, y2) touch the parabola at
(x1, y1) and (x3, y3), then
(A) x1, x2, x3 are in A.P. (B) x1, x2, x3 are in G.P.
(C) y1, y2, y3 are in A.P. (D) y1, y2, y3 are in G.P.

x y x2 y 2
32. If the line   1 is normal to the ellipse 2  2  1 , then possibly
a b  
(A)  > a,  > b (B)  < a,  < b
(C)  > a,  < b (D)  < a,  > b

33. The coordinates of the point on the ellipse 4x2 + 9y2 = 1 at which the tangent to the
ellipse is parallel to the line 8x = 9y are
 2 1  2 1
(A)  ,  (B)   , 
5 5  5 5
 2 1  2 1
(C)   ,   (D)  ,  
 5 5 5 5
x 2
y2
34. From the point (h, k), normals are drawn to the ellipse 2  2  1 . The feet of these
a b
normals lie on a hyperbola one of whose asymptotes is
(A) (a2  b2) x  a2h = 0 (B) (a2  b2) x + a2h = 0
2 2 2
(C) (a  b ) y  b k = 0 (D) (a2  b2) y + b2k = 0

x2 y2
35. The tangent and the normal to the ellipse  1 at the point (h, k) meet the y-axis

a 2 b2
at A and B respectively. The circle with AB as diameter, passes through
(A) ( a, 0) (B) (h, k)
(C) ( ae, 0) (D) (0,  b)

FIITJEE Ltd., ICES House, 29-A, Kalu Sarai, Sarvapriya Vihar, New Delhi -110016, Ph 26515949, 26569493, Fax 26513942
FIITJEE Ltd. Material Provided by - Material Point Available on - Learnaf.com
M ast er A ssignm ent s P-57-P8-CBSE-MA-64

x2 y2
36. The value of m for which y = mx + 6 is a tangent to the hyperbola  = 1 is
100 49
17 20
(A) (B)
20 17
20 17
(C)  (D) 
17 20

37. Straight line Ax + By + D = 0 would be tangent to xy = c2, if


(A) A > 0, B > 0 (B) A < 0, B < 0
(C) A > 0, B < 0 (D) A < 0, B > 0

38. Let an be product of first n natural numbers. Then for all n  N


n
n  n  1
(A) n  an (B)    n!
 2 
(C) nn  an+1 (D) none of these

 8
39. If 1, log3 (3x  2), 2 log9  3x   are in A.P., the value of x can be
 3
 4
(A) 1 (B) log  
3
log 4
(C) (D) log3 4
log3

x 2  ax  3
40. If takes all real values for possible real values of x, then
x2  x  a
(A) 4a3 + 39  0 (B) 4a3 + 39 < 0
(C) a < 1/4 (D) a  1/4


41. If a complex number z has argument and modulus unity, then z2 + z
3
(A) is purely imaginary (B) has modulus 3
(C) is purely real (D) none of these

42. If the polynomial expression x3 + ax2 + bx + c is divisible by (x + 1)2, then


(A) a  b + c = 1 (B) 2a  b = 3
(C) a+ b + c =  1 (D) 2a  b =  3

43. If (p1, q1, r1) be the image of (p, q, r) in the plane ax + by + cz + d = 0, then
p  p q1  q r1  r
(A) 1   (B) a(p + p1) + b(q + q1)+c(r + r1)+2d = 0
a b c
p  p q1  q r1  r
(C) 1   (D) none of these
a b c

44. The equation of the plane bisecting the angle between the planes 2x – y + 2z + 3 = 0 and
3x – 2y + 6z + 8 = 0 is
(A) 23x – 13y + 32z + 45 = 0 (B) 5x – y – 4z = 3
(C) 5x – y – 4z + 45 = 0 (D) 23x – 13y + 32z + 3 = 0

FIITJEE Ltd., ICES House, 29-A, Kalu Sarai, Sarvapriya Vihar, New Delhi -110016, Ph 26515949, 26569493, Fax 26513942
FIITJEE Ltd. Material Provided by - Material Point Available on - Learnaf.com

P-57-P8-CBSE-MA-65 Pinnacle Study Package

45. If the plane ax + by + cz + d = 0 intersects the x and the y-axes at equal angles, then
(A) |a| = |b| (B) a = –b
2 2
(C) a = b (D) a + b = 1
x  2 y 1 z 1
46. The line   intersects the curve xy = c2, z = 0 if c is equal to
3 2 1
1
(A) 1 (B)
3
1
(C) 5 (D) 
3
47. The locus of a point which is equidistant from the x-axis and the y-axis is
(A) the plane x = y (B) the plane x = –y
(C) the plane x + 2y = 0 (D) the plane x  2y = 0

48. A and B are two independent events. If the probability that A and B happen
simultaneously is 1/12 and neither A nor B happens is 1/2, then
(A) P(A) = 1/3, P(B) = 1/4 (B) P(A) = 1/2, P(B) = 1/6
(C) P(A) = 1/4, P(B) = 1/3 (D) P(A) = 1/6, P(B) = 1/2

49. If the sum of the arguments of the roots of the equation zn = k (n  N) has principal value
0, then
(A) k  0 and n is even (B) k  0 and n is odd
(C) k  0 and n is even (D) k  0 and n is odd

50. If |z1 + z2|  |z1  z2|, then


 z   z 
(A)   arg  1   (B) Re  1   0
2  z2  2  z2 
z  z 
(C) Re  1   0 (D) Im  1   0
 z2   z2 

51. If f(x) = |x  2| 2  x  3
= [x] 3 < x  6,
then
(A) lim f  f(x)   1, lim f  f(x)   1 (B) lim f  f(x)  1
x 3 x 3 x 3
(C) lim f  f(x)   1 (D) lim f  f(x)  1
x 3 x 3

3 5
log2 x 2 log2 x 
52. The equation x 4  2 has 4

(A) at least one real root (B) exactly three real solutions
(C) exactly one irrational solution (D) complex roots

53. The value of  satisfying the equation [sin(cot–1 cot tan–1 x)] = sin , (where x  R and [.]
denotes the greatest integer function) is

(A) 0 (B)
2
3
(C)  (D)
2

FIITJEE Ltd., ICES House, 29-A, Kalu Sarai, Sarvapriya Vihar, New Delhi -110016, Ph 26515949, 26569493, Fax 26513942
FIITJEE Ltd. Material Provided by - Material Point Available on - Learnaf.com
M ast er A ssignm ent s P-57-P8-CBSE-MA-66

  
54. If a, b, c are three non-coplanar vectors such that volume of the parallelopiped formed
  
with a, b, c as co-terminous edges is equal to volume of the parallelopiped formed with
     
a  b, b  c, c  a as co-terminous edges, then
     
(A) a, b, c   0 (B) a, b, c   1
     
(C) a, b, c   1 (D) a, b, c    1, 1

55. Let AC and BD be two chords of a circle with centre O such that they intersect at right
angles inside the circle at point M. Suppose K and L are the mid points of the chords AB
and CD respectively, then
(A) OK = LM (B) OKLM is a parallelogram
(C) OKML is a trapezium (isosceles) (D) none of these

10
56. If f (x)  f(x)   f(x)dx , then
10
(A) f(x) = f(x) if lim f(x)  0 (B) f(x)  f(x) = c (any const.)
x 
(C) f(x)  f(x) = c (any const) (D) none of these.

57. If a, b and c are rational numbers (a > b > c > 0) and the quadratic equation
(a + b  2c)x2 + (b + c  2a)x + (c + a  2b) = 0 has a root in the interval (1, 0), then
(A) c + a < 2b
(B) Both roots are rational
(C) the equation ax2 + 2bx + c = 0 have both negative real roots
(D) the equation cx2 + 2ax + b = 0 have both negative real roots.

n n
r n2  3n  3
58. If  Cr
r 0
n 

r 0 2 n Cr
, then

(A) n = 1 (B) n = 2
(C) n = 3 (D) none of these.

59. A tangent, drawn to the curve y = f(x) at P(x, y), cuts the x-axis and the y-axis at A and B
respectively. If BP : AP = 3 : 1 and f(1) = 1, then
dy
(A) equation of the curve is x  3y  0
dx
(B) normal to the curve at (1, 1) is x + 3y = 4
 1
(C) the curve passes through the point  2, 
 8
dy
(D) differential equation of the curve is x  3y  0
dx

60. Let A be a vector paralle to the line of intersection of planes P1 and P2 through the
origin. If P1 is parallel to vectors 2iˆ  3kˆ and 4iˆ  3kˆ , and P2 is paralle to vectors ˆj  kˆ

and 3iˆ  3ˆj , then the angle between A and vector 2iˆ  ˆj  2kˆ is
 
(A) (B)
2 4
 3
(C) (D)
6 4

FIITJEE Ltd., ICES House, 29-A, Kalu Sarai, Sarvapriya Vihar, New Delhi -110016, Ph 26515949, 26569493, Fax 26513942
FIITJEE Ltd. Material Provided by - Material Point Available on - Learnaf.com

P-57-P8-CBSE-MA-67 Pinnacle Study Package

ANSWERS

1. A, B, C 2. B, C 3. A, B
4. A, C 5. A, B 6. A, C
7. A, C 8. B, C 9. A, B
10. A, C 11. B, C 12. B, C
13. A, B, C 14. A, B 15. A, B, D
16. A, D 17. A, C, D 18. B, C
19. B, D 20. B, C 21. A, D
22. A, B, C, D 23. A, D 24. B, D
25. A, B 26. B, D 27. A, B
28. C, D 29. A, D 30. A, C
31. B, C 32. A, C, D 33. B, D
34. A, D 35. B, C 36. A, D
37. A, B 38. A, B 39. A, C, D
40. B, C 41. A, B 42. A, B
43. A, B 44. A, B 45. A, C
46. B, D 47. A, B 48. A, C
49. B, C 50. A, B 51. B, C, D
52. A, B, C 53. A, C 54. B, C
55. A, B 56. A, B, C 57. A, B, C, D
58. A, C 59. C, D 60. B, D

FIITJEE Ltd., ICES House, 29-A, Kalu Sarai, Sarvapriya Vihar, New Delhi -110016, Ph 26515949, 26569493, Fax 26513942
FIITJEE Ltd. Material Provided by - Material Point Available on - Learnaf.com
M ast er A ssignm ent s P-57-P8-CBSE-MA-68

MATCHING

Match the correct answer in each of the following cases

   
1. Let f: (1, 1)   ,  be a function
 4 4
2
1
(i) f(x) = tan1 2  1 (A) f is oneone but not onto
2
x
d  t 
 sin1 dt 
(ii) f(x) = 
dx 
0
2 

(B) f is onto but not oneone

(C) f is both oneone and onto


2 2 2
2. Consider the equation (13x  1) + (13y  2) = k(5x  12y + 1) .
(i) k = 1 (A) a pair of straight lines
(ii) k = 169 (B) a parabola
(C) an ellipse
(D) a hyperbola
3. Let f(x) = x(x + 1)(x + 2)(x + 3).
(i) x = 3/2 (A) local maximum
(ii) x = 1 (B) local minimum
(C) not a critical point
4. z is a non-zero complex number
(i) z = z (A) Re z2 = 0

(ii) arg z = (B) Re z3 = 0
6
(C) Im z3 = 0
 p 1
x sin  x x , x0
5. Let f(x) =  x

0 x 0
(i) p = 1 (A) f(x) is differentiable at all point except x = 0
(ii) p = 2 (B) f(x) exists at all points
(C) f(x) exists but is not continuous at x = 0
6. z ( 0) is a complex number
(i) Re z= 0 (A) Re z2 = 0
 2
(ii) Arg z = (B) Im z = 0
4
(C) Re z2 = Im z2
sin3
7. is
cos 2
 13 14 
(i) positive (A)    , 
 48 48 
 14 18 
(ii) negative (B)    , 
 48 48 
 18 23 
(C)    , 
 48 48 
 
(D)    0, 
 2

FIITJEE Ltd., ICES House, 29-A, Kalu Sarai, Sarvapriya Vihar, New Delhi -110016, Ph 26515949, 26569493, Fax 26513942
FIITJEE Ltd. Material Provided by - Material Point Available on - Learnaf.com

P-57-P8-CBSE-MA-69 Pinnacle Study Package

   
8. Let the function defined have the domain  , 
 2 2
(i) x + sin x (A) increasing
(ii) sec x (B) decreasing
(C) neither increasing nor decreasing

9. The function y = f(x)


(i) f(x) = sin ([x]) (A) differentiable everywhere
(ii) f(x) = sin ((x  [x])) (B) nowhere differentiable
(C) not differentiable at x = 1 and  1.
   
10. Function f(x) has domain  ,  and range (,).
 2 2
(i) f(x) = 1 + 2x (A) onto but not oneone
(ii) f(x) = tan x (B) one-one but not onto
(C) oneone and onto
(D) neither oneone not onto

11. The value of


(i) sin 12 sin 48 sin 54 1
(A)
8

(ii) y-coordinate of the point of (B) 5


contact of y = x  1 with
3x2  4y2 = 12

(iii) r, when n1Cr 1 :nC2 :n1Cr 1 (C) 3


= 11 : 6 : 3

12. The value of t when


(i) 7 and 2 are roots of (A) 2
t 3 7
2 t 2  0
7 6 t
(ii)   [0, 2], and t2  4t + 5  sin  = 0 (B) 6
31 (C) 9
(iii) in triangle ABC, cos (A  B) = , a = 5, b
32
= 4, c = t

13. Numerical value of


(i) x at the point of maximum of 8
x (A)
3 2 2 3
 2  t  1 t  2
1
 3  t  1  t  2  dt
2 4 (B) 1
(ii)  f  x  dx when  f  x  dx  4 and
1 1
4

  3  f  x   dx  7
2
2
(iii) The area bounded by y = x  2x + 3, the (C) 5
tangent to it at P(2, 5) and the y  axis

FIITJEE Ltd., ICES House, 29-A, Kalu Sarai, Sarvapriya Vihar, New Delhi -110016, Ph 26515949, 26569493, Fax 26513942
FIITJEE Ltd. Material Provided by - Material Point Available on - Learnaf.com
M ast er A ssignm ent s P-57-P8-CBSE-MA-70

14. Required
(i) Number of points inside or on (A) 32
x2 + y2 = 4 subject to
4 4 2
cot x + tan x + 1 = 3 sin y.
(ii) Least value of a when (B) 4
x = a + m, y = 2, y = mx are
con-current lines
(iii) Area of the rectangle ABCD, (C) 2 2
when A is (3, 4), B is (5, 4) and
one of the diameters of the circle
circumscribing rectangle ABCD is
4y = x + 7.
2
15. Normals with feet at P, Q, R are drawn from the point (3, 0) to the parabola y = 4x. The
triangle PQR is such that its
(i) area (A) 2
(ii) circum-radius (B) 5/2
(iii) centroid (C) (5/2, 0)
(iv) circumcentre (D) (2/3, 0)

16. The vertices of a triangle ABC lie on a rectanglar hyperbola such that its orthocentre is at
(3, 2). If the asymptotes of the hyperbola are parallel to the coordinates axes and two of
its tangents from the point (1, 1) are perpendicular, then
(i) the equation of the asymptotes is (A) 2
(ii) the equation of the hyperbola is (B) xy = x + y + 1
(iii) number of real normals that can be drawn to the hyperbola (C) 3
from the point (1, 1) is
(iv) the ratio of the portion of the intercept of the asymptotes (D) xy + 1 = x + y
and the hyperbola by a line is k : 2 where k equals

17. The sides AB and BC of a triangle ABC are members of the family of lines ax + by + c =
0, and a, b, c are in A.P. If the orthocentre of the triangle is at (2, 4), and its vertex A at
(1, 1), then
(i) the vertex B is at (A) (5, 1)
 17 1
(ii) the vertex C is at (B)   ,  
 2 2
(iii) the centroid of triangle ABC is at (C) (17, 4)
(iv) the circumcentre of triangle ABC is at (D) (1, 2)

18. From the vertices A, B, C of the triangle ABC, altitudes AD, BE and CF are drawn to the
opposite sides and triangle DEF is completed. With usual notations for triangle ABC,
R
(i) ratio of the areas of triangles DEF and ABC is (A)
2
(ii) circumradius of triangle DEF is (B) 2R cosA cosB cosC
r
(iii) in-radius of triangle DEF is (C)
R
(iv) ratio of the perimeters of triangles DEF and ABC is (D) 2 cosA cosB cosC

FIITJEE Ltd., ICES House, 29-A, Kalu Sarai, Sarvapriya Vihar, New Delhi -110016, Ph 26515949, 26569493, Fax 26513942
FIITJEE Ltd. Material Provided by - Material Point Available on - Learnaf.com

P-57-P8-CBSE-MA-71 Pinnacle Study Package

ANSWERS
1. (i)  C (ii)  A
2. (i)  B (ii)  D
3. (i)  A (ii)  C
4. (i)  C (ii)  B
5. (i)  A (ii)  C
6. (i)  B (ii)  A
7. (i)  C (ii)  A
8. (i)  A (ii)  C
9. (i)  A (ii)  C
10. (i)  B (ii)  C
11. (i)  A (ii)  C (iii)  B
12. (i)  C (ii)  A (iii)  B
13. (i)  B (ii)  C (iii)  A
14. (i)  B (ii)  C (iii)  A
15. (i)  A (ii)  B (iii)  D (iv)  C
16. (i)  C (ii)  B (iii)  D (iv)  A
17. (i)  C (ii)  D (iii)  B (iv)  A
18. (i)  B (ii)  C (iii)  D (iv)  A

FIITJEE Ltd., ICES House, 29-A, Kalu Sarai, Sarvapriya Vihar, New Delhi -110016, Ph 26515949, 26569493, Fax 26513942
FIITJEE Ltd. Material Provided by - Material Point Available on - Learnaf.com
M ast er A ssignm ent s P-57-P8-CBSE-MA-72

COMPREHENSION

Read the following concepts along with the illustrations carefully and answer the exercises
accordingly:

I. Concept: A circle is the locus of a point in a plane such that its distance from a fixed
point in the plane is constant. Anologously, a sphere is the locus of a point in space such that its
distance from a fixed point in space is constant. The fixed point is called the centre and the
constant distance is called the radius of the circle/sphere. In anology with the equation of the
  
circle |z  c| = a, the equation of a sphere of radius a is | r  c | = a, where c is the position

vector of the centre and r is the position vector of any point on the surface of the sphere. In
Cartesian system, the equation of the sphere, with centre at ( g,  f,  h) is
x2 + y2 + y2 + 2gx + 2fy + 2hz + c = 0 and its radius is f 2  g2  h2  c .

Illustration : Radius of the sphere, with (2,  3, 4) and ( 5, 6,  7) as extremities of a


diameter, is
251 251
(A) (B)
2 3
251 251
(C) (D)
4 5

Solution: The centre of the sphere is at the mid-point of the extremities of a diameter
2 2 2
 3 3 3 7 9  11
 the centre is   , ,   and thus the radius =         .
 2 2 2 2 2 2
Hence (C) is the correct answer.

Exercise 1
(i). The centre of the sphere (x  4) (x + 4) + (y  3) (y + 3) + z2 = 0 is
(A) (4, 3, 0) (B) ( 4,  3, 0)
(C) (0, 0, 0) (D) none of these
(ii). Equation of the sphere having centre at (3, 6,  4) and touching the

 
plane r  2iˆ  2ˆj  kˆ  10 , is (x  3) + (y  6) + (z + 4) = k , where k is equal to
2 2 2 2

(A) 3 (B) 4
(C) 6 (D) 17

(iii). Radius of the circular section of the sphere | r |  8 cut off by the plane
 ˆ
 
r  i  2ˆj  2kˆ  15 is
(A) 39 (B) 29
(C) 59 (D) 7

II. Concept: The CIRCULAR MEASURE of an angle is defined as the ratio of the
length of the arc by which it is subtended at the centre of the circle to the length of the
radius of that circle. The angle subtended by an arc equal in length to the radius at the
centre of the circle is called a radian (1c) so that
c c
2 = 360 i.e.  = 180.

Three important results associated with circular measure follow:

FIITJEE Ltd., ICES House, 29-A, Kalu Sarai, Sarvapriya Vihar, New Delhi -110016, Ph 26515949, 26569493, Fax 26513942
FIITJEE Ltd. Material Provided by - Material Point Available on - Learnaf.com

P-57-P8-CBSE-MA-73 Pinnacle Study Package

(a) The length of an arc which subtends an angle of c at the centre of a circle of radius

r is  2r  r .
2
c
(b) The area of a sector of angle  of a circle of radius r is
 1
 r 2  r 2  .
2 2

(c) Suppose PQ is an arc of a circle, radius r, P


subtending angle c (< /2) at the centre O and
the tangent at P meets OQ produced at T.  T
Then area of OPT > area of sector OPQ > area of O Q
OPQ
1 1 1
i.e. r 2 tan   r 2   r 2 sin 
2 2 2
 tan >  > sin ( < /2).

Illustration: P is a fixed point on a circle of unit radius whose centre is O. A chord


perpendicular to OP at a distance x from P cuts the circle at A and B. Then the
length of the arc AB, if 0 < x < 1, is equal to
(A) cos1x (B) 2cos1x
1
(C) cos (1  x) (D) 2cos1(1  x)

Solution: Let PAB = . Then cos = 1  x P


  = cos1(1  x). A x
The arcAB = 2 = 2 cos1(1  x). B
Hence (D) is the correct answer. 1x

O

Exercise 2
(i). Area of the sector OAB in the illustration, if 0 < x < 1, is
(A) 2cos1x (B) cos1x
1
(C) 2cos (1  x) (D) cos1(1  x).
(ii). Area of the triangle OAB in the previous question, if 0 < x < 1, is
(A) 1  x  x  2  x  (B) sin{2cos1(1  x)}
1 1
(C)
2

cos 2 cos 1 1  x   2
sin 2 cos 1 x .
(D)  
1
(iii). The area of the segment of circle APBA, when x  is
2
 3 2 3
(A)  (B) 
3 4 3 4
 3 2 3
(C)  (D) 
3 2 3 2

FIITJEE Ltd., ICES House, 29-A, Kalu Sarai, Sarvapriya Vihar, New Delhi -110016, Ph 26515949, 26569493, Fax 26513942
FIITJEE Ltd. Material Provided by - Material Point Available on - Learnaf.com
M ast er A ssignm ent s P-57-P8-CBSE-MA-74

III. Concept: If a circle with centre C(, ) intersects a rectangular hyperbola with
centre L(h, k) at four points P(x1, y1), Q(x2, y2), R(x3, y3) and S(x4, y4), then the mean of
the four points P, Q, R, S is the mean of the points C and L. In other words, the midpoint
of CL coincides with the mean point of P, Q, R, S. Analytically,
x1  x2  x3  x 4   h y1  y2  y3  y 4   k
 ;  .
4 2 4 2

Illustration: Five points are selected on a circle of radius a. Prove that the centres of the
rectangular hyperbolas, each passing through four of these points, all lie on a
circle of radius a/2.
2 2 2
Solution: Let the circle be x + y = a and let the centre of a rectangular hyperbola be
(h, k). Let the given points on the circle be (acosi, asini), i = 1, 2, 3, 4, 5… so
that
acos 1  a cos 2  acos 3  a cos  4 0  h h
 
4 2 2
5
  acos   acos 
i1
i 5  2h .

5
Similarly,  a sin   a sin 
i1
i 5  2k .

5 5
Let 
i1
acos i  L and  a sin   M .
i1
i

Since the points are given, L and M are known. Hence we have
acos5 = L  2h, asin5 = M  2k.
 a2 = (2h  L)2 + (2k  M)2 and hence (h, k) lies on the curve
2 2 2
 L  M a a
 x     y      which is a circle with radius .
 2   2  2
  2

Exercise 3
(i) A, B, C and D are the points of intersection of a rectangular hyperbola and a circle.
If AB passes through the centre of the hyperbola, then centre of the circle lies at
the mid-point of
(A) BC (B) CA
(C) CD (D) none of these
(ii) A series of circles is drawn having for its diameters the asymptotes of a
rectangular hyperbola. Two of the common chords of the hyperbola and the circle
are parallel to
(A) the line joining a vertex and the end of a latus rectum of the hyperbola
(B) the line joining the centre and the end of a latus rectum of the hyperbola.
(C) the line joining the ends of the two latus recta
(D) none of these
(iii). A circle touches a rectangular hyperbola at two points P and Q. The line PQ passes
through
(A) the centre of the circle
(B) the centre of the hyperbola
(C) the mid-point of the centre of the circle and the centre of the hyperbola
(D) none of these

FIITJEE Ltd., ICES House, 29-A, Kalu Sarai, Sarvapriya Vihar, New Delhi -110016, Ph 26515949, 26569493, Fax 26513942
FIITJEE Ltd. Material Provided by - Material Point Available on - Learnaf.com

P-57-P8-CBSE-MA-75 Pinnacle Study Package

IV. Concept:

2k 2k
The n nth roots of unity are cos  i sin , (k = 1, 2, ……, n – 1), so that for n odd
n n
n 1
2
 2k 2k  2k 2k 
x n  1   x  1   x  cos  i sin  x  cos  i sin
k 1
n n  n n 
n 1
2

or x n / 2  x n / 2  x1/ 2  x 1/ 2    x  x1  2cos 2nkx 
k 1
and hence writing x = cos2 + isin2, we get
n 1
n 1 2
 2k 
sin n  2 2 sin  
cos 2
k 1
 cos
n ,

(n odd). ….(1)

Similarly for n even


n2
n 2 2
 2k 
sin n  2 2 sin 2   cos 2  cos
k 1
n .

….(2)

 2 3 4 1
Illustration: Prove that cos cos cos cos  .
5 5 5 5 16

Solution: We take n = 5 (odd).


2
  2k 
Writing  =
2
in (1), we get 1 = 4
k 1 

 1  cos 5 

2
k  2
or, 1 = 4  2cos
k 1
2
5
 4.2cos2 .2cos2
5 5
 2  2  2  4  3 
= 16 cos cos cos cos = 16 cos cos   cos   cos 
5 5 5 5 5 5  5  5 
 2 3 4
= 16 cos cos cos cos .
5 5 5 5

Alternative Solution:
We take n = 10 (even).

Writing  = in (2), we get
4
4
 k 
1  24    cos 5 
k 1
 2 3 4
 1  24 cos cos cos cos
5 5 5 5
 2 3 4 1
 cos cos cos cos  .
5 5 5 5 16

FIITJEE Ltd., ICES House, 29-A, Kalu Sarai, Sarvapriya Vihar, New Delhi -110016, Ph 26515949, 26569493, Fax 26513942
FIITJEE Ltd. Material Provided by - Material Point Available on - Learnaf.com
M ast er A ssignm ent s P-57-P8-CBSE-MA-76

Exercise 4
 2 3
(i) The value of tan tan tan is
7 7 7
1 3
(A) (B)
7 7
5
(C) (D) 7
7
   2  3 
(ii) If sin7 = k sin  sin2  sin 2    sin 2  sin 2   sin2  sin2   then k is
 7  7  7 
equal to
(A) 1 (B) 64
1
(C) 32 (D)
64
m 1
sin m  k 
(iii). If  2p   cos  cos m  , then p is equal to
sin k 1
(A) m (B) m – 1
(C) m – 2 (D) m + 1

  
V. Concept: Let a, b, c be a system of three noncoplanar vectors. Then the system
        
of vectors a, b, c  which satisfies a  a  b  b  c  c   1 and
           
a  b  a  c   b  a  b  c   c   a  c   b  0 is called the reciprocal system to the vectors
     
       bc  ca  a b
a, b, c. In terms of a, b, c. , a  , b  , c 
a b c  a b c  a b c 
1 0 0
     
and a b c  a b c   0 1 0  1 .
0 0 1

  
Illustration: If the vectors b, c, d are not coplanar, then prove that
            
         
a  b  c  d   a  c   d  b  a  d  b  c is parallel to a .

  
Solution: Let b, c , d be the reciprocal system to the given vectors.
           
        
Then a  b  c  d   a  c   d  b  a  d  b  c 
           
   
= b c d  a  b  b   a  c   c   a  d  d
 
              
   
= b c d  a  b  b  a   a  c    c  a  a  d  d  a 
 
           
= b c d    
     
 
    a  b  c   d  a  c    d  b a  d  b  c 
   

 3a 
 a b c  a b c  a b c  
       
        
= b c d a  3a  2 b c d a

which is parallel to vector a .

FIITJEE Ltd., ICES House, 29-A, Kalu Sarai, Sarvapriya Vihar, New Delhi -110016, Ph 26515949, 26569493, Fax 26513942
FIITJEE Ltd. Material Provided by - Material Point Available on - Learnaf.com

P-57-P8-CBSE-MA-77 Pinnacle Study Package

Exercise 5
     
(i). If vectors a, b, c and a  , b  , c  from reciprocal systems to each other, then
     
a  a   b  b  c  c  is equal to
     
(A) a  b  c (B) a   b   c 

(C) 0 (D) None of these.
     
(ii).  b  c c  a a  b  is equal to
 
      2
(A)  a b c  (B)  a b c 
  
(C) 2  a b c  (D) 0
  
(iii). If a, b and c represent the three concurrent edges of a tetrahedron, the length of
whose edges are 4, 3 and 2 respectively then the value of
       
 
a  b  c  ab  bc  ca is equal to 
(A) 0 (B) 8
(C) 12 (D) 6

(iv). Any vector r can be represented as
                 
 
(A)  r  a  a  r  b b   r  c  c  
(B)  r  a   a  r  b  b   r  c   c
     
    
(C) r  iˆ a   r  ˆj b   r  kˆ c  (D) none of these

VI. Concept: Locus of a point, which moves in such a way that ratio of its distance
from a fixed point to its distance from a fixed line is constant (e), is a conic: It is parabola
when e = 1, ellipse when e < 1 and hyperbola when e > 1. Also locus of a point which
moves in such a way that sum of its distances from two fixed points is constant and is
more than the distance between the fixed points is an ellipse. Locus of a point which
moves in such a way that difference of distance from two fixed points is constant and is
less than the distance between the fixed points, is a hyperbola.

Illustration: Let C1 and C2 be two circles touching each other externally. A circle C encircles
both i.e. C1 and C2 touch C internally. Locus of the centre of C is
(A) Hyperbola (B) Parabola
(C) Ellipse (D) None of these.

Solution: Let C1(O1, r1) and C2(O2, r2) be two circles. They touch each other externally.
O1O2 = r1 + r2.
Let the circle C(O, r) touch both C1 and C2 then OO1 = r  r1, OO2 = r  r2.
 OO2  OO1 = r1  r2 < r1 + r2
Hence the locus of O is a hyperbola with focii O1, O2.

FIITJEE Ltd., ICES House, 29-A, Kalu Sarai, Sarvapriya Vihar, New Delhi -110016, Ph 26515949, 26569493, Fax 26513942
FIITJEE Ltd. Material Provided by - Material Point Available on - Learnaf.com
M ast er A ssignm ent s P-57-P8-CBSE-MA-78

Exercise 6

(i). Let C1 and C2, be two circles C1 containing C2 inside it. A circle C, lying inside C1,
touches C1 internally and C2 externally. Locus of the centre of C is
(A) Hyperbola (B) Parabola
(C) Ellipse (D) None of these.

(ii) A point P(x, y) moves in such a way that


2 2 2
x  2   y  1 
2
 x  2   y  2 2 1   a . The locus of P is
(A) hyperbola if a < 4 (B) ellipse if a > 4
(C) parabola if a = 4 (D) none of these

2
(iii). A point P moves in such a way that  x  1
2
 y2   x  1
2

 y  12   b . The
locus of P(x, y) is
(A) hyperbola if b = 4 (B) ellipse if b = 4
(C) parabola if b = 4 (D) none of these


n1  x
VII. Concept: For n > 0, the integral x e dx , which is a function of n, is called
0

n1  x
gamma function and is denoted by n . Thus x e dx  n .
0
  
2  5
For example  x3 e x dx  x 4 1e x dx  4 and
 x
2 / 3 x
e dx    1  .
0 0 0
3  3

1
For n > 1, n  n  1 n  1 and  .
2


 x3
Illustration: The value of e dx is
0

5 4
(A) (B)
3 3
2
(C) (D) none of these
3

1 2 / 3
Solution: Let x3 = t , so that x = t1/3 and dx = t dt.
3
  2
3 1  3 t
Hence  e x dx =  t e dt
3
0 0
 1
1 3 1  t 1 1 4  4  4
= 
t e dt  =   1   1    .
3 3 3 3  3  3
0
Hence (B) is the correct answer.

FIITJEE Ltd., ICES House, 29-A, Kalu Sarai, Sarvapriya Vihar, New Delhi -110016, Ph 26515949, 26569493, Fax 26513942
FIITJEE Ltd. Material Provided by - Material Point Available on - Learnaf.com

P-57-P8-CBSE-MA-79 Pinnacle Study Package

Exercise 7

3  x3
(i) The value of x e
0
dx is

1  1 1 4
(A)   (B)  
3 3 9 3
1  1
(C)   (D) none of these
9 3

3
(ii) The value of  x e  x dx is
0
1 1
(A)  (B) 
2 3
1
(C)  (D) none of these
4

a2 x 2
(iii) The value of e
0
dx is

 
(A) (B)
2a 2a2

(C) (D) none of these
a

VIII. Concept: The volume of the solid generated by the revolution of the area
bounded by the curve y = f(x), the x-axis and the ordinates x = a, x = b, about the x-axis,
b
2
is  y dx , where y = f(x) is a finite, continuous and single-valued function of x in the
a
interval a  x  b.
The volume of the solid generated by the revolution about the y-axis of the area
b
2
bounded by the curve x = f(y), the y-axis and the abscissae y = a and y = b is  x dy .
a

Illustration: The volume of solid generated by rotating the area, bounded by the
coordinates axes and the curve y = cosx, about the y-axis is
 
(A)    2 (B)    2
2 2
(C) ( 2) (D) ( + 2)

Solution: The equation of the curve is y = cosx. This meets the coordinates axes at
 
(0, 1) and  , 0  . Since the curve is rotating about the y-axis, the volume
2 
of the solid thus generated

FIITJEE Ltd., ICES House, 29-A, Kalu Sarai, Sarvapriya Vihar, New Delhi -110016, Ph 26515949, 26569493, Fax 26513942
FIITJEE Ltd. Material Provided by - Material Point Available on - Learnaf.com
M ast er A ssignm ent s P-57-P8-CBSE-MA-80

1 /2 /2
2 2dy 2
=  x dy   x dx =  x   sin x  dx
dx
0 0 0

 /2  /2 
/2 /2
=    x2   cos x 0 2x cos x dx  = 2  x  sin x 
  0
 sin x dx  
   
 0   0 
 / 2   
= 2   cos x 0   2   1 = ( + 2).
2  2 
But  > 2 and thus volume =  (  2).
Hence (C) is the correct answer.

Exercise 8
a
(i) The volume of a spherical cap of height cut off from a sphere of radius a is
2
5a3 5a3
(A) (B)
12 24
7a3 7a3
(C) (D)
24 12
(ii) The volume of the reel – shaped solid formed by the revolution about the y-axis of
the part of the parabola y2 = 4ax cut off by the latus rectum is
4 4
(A) a3 (B) a3
5 3
2 2
(A) a3 (D) a3
3 5
(iii) The figure bounded by the parabola y2 = 4ax and the tangents at the extremities of
its latus rectum revolves about the axis of the parabola. The volume of the surface
thus generated is
4 3 2
(A) a (B) a3
3 3
5 3
(C) a (D) a3
3

IX. Concept: The quadratic function y = ax2 + bx + c, a, b, c  R, can be written


2
 b   4ac  b2 
as y = a  x      which represents a family of parabolas. The vertex of this
 2a   4a 
 b 4ac  b2   b 4ac  b2  1  1
parabola is   ,  , with focus at   ,  and latus rectum .
 2a 4a   2a 4a  4 a

Illustration : If the tangents drawn from the point (1, a) to the parabola y = ax2 + bx + c
are perpendicular to each other, then the roots of the equation ax2 + bx +
c =0 are
(A) real and distinct (B) real and equal
(C) imaginary (D) real and distinct only if a > 0.

FIITJEE Ltd., ICES House, 29-A, Kalu Sarai, Sarvapriya Vihar, New Delhi -110016, Ph 26515949, 26569493, Fax 26513942
FIITJEE Ltd. Material Provided by - Material Point Available on - Learnaf.com

P-57-P8-CBSE-MA-81 Pinnacle Study Package

b  4ac  b2  a
Solution: The line x + = M  y   + is a tangent to the given
2a  4a  4M
parabola. Since it passes through (1, a),
b  4ac  b2  a
1+ = M  a   +
2a  4a  4M
or (4a2 + b2  4ac)M2  2M (2a + b) + a2 = 0.
Since the two tangent are perpendicular to each other.
1 4a2  b2  4ac
 1   1 or b2  4ac =  5a2
M1M2 a2
 b2  4ac < 0.
Hence (C) is the correct answer.

Alternative:
Since the tangents are perpendicular to each other, the point (1, a) must
4ac  b2  1
lie on the directrix y = of the parabola.
4a
4ac  b2  1 2 2
Hence  a  b  4ac =  (1 + 4a ) < 0.
4a

Exercise 9
(i) For fixed values of a and b and variable c, the family of parabolas y = ax2 + bx + c
has
(A) a common axis (B) a common focus
(C) a common directix (D) none of these
(ii) For fixed values of b and c and variable a, the family of parabolas y = ax2 + bx + c
(A) are equal parabolas (B) passes through a common point
(C) touches a common line (D) none of these
(iii) The tangent at the point (1, 1) to the parabola y = ax2 + bx + c meets the
perpendicular to it from the focus of the parabola at the point (0, a). Then
(A) b = 0 (B) b < 1
(C) b = 1 (D) b > 1

X. Concept:

A point P(x1, y1) lies inside, outside or on the curve S = 0, if S1 < 0, S1 > 0 or S1 = 0
respectively. In case S = 0 represents a straight line, two points P(x, y1) and Q(x2, y2) lie
on the same side of the line if S1 and S2 have the same sign. If S1 and S2 have opposite
signs, P and Q will be lying on the opposite sides of S = 0.

Illustration: If (, ) is a point of discontinuity of the function y = f(f(f(x))), where f(x) =


1
, then the values of a for which the points (, ) and (a, a2) lie on the
1 x
same side of the line x + 2y  3 = 0 belong to
3 3
(A)   a 1 (B)   a 1
2 2
3
(C) a  1 (D) a  
2

FIITJEE Ltd., ICES House, 29-A, Kalu Sarai, Sarvapriya Vihar, New Delhi -110016, Ph 26515949, 26569493, Fax 26513942
FIITJEE Ltd. Material Provided by - Material Point Available on - Learnaf.com
M ast er A ssignm ent s P-57-P8-CBSE-MA-82

Solution: We note that f(x) is discontinuous at x = 1.


1 1 x
Moreover, f(f(x)) =  is discontinuous at x = 1 and x = 0.
1 x
1
1 x
1
1
Hence f(f(f(x)) = 1 x  x i.e. y = x is discontinuous at (0, 0) and
1

1 x
(1, 1).
The point (1, 1) lies on the given line x + 2y  3 = 0. For (0, 0), S1 < 0
and hence for (a, a2), S1 < 0
 a + 2a2  3 < 0 or (2a + 3) (a  1) < 0.
Hence (A) is the correct answer.

Exercise 10
(i) If the points (7, 5) and (, 2) lie on the opposite sides of the line 5x  6y  1 = 0,
then a belongs to
(A) [-0, 1] (B) (1, 1)
 1 1 
(C)  ,  (D) (2, 4)
3 2
(ii) The circle x2 + y2  6x  10y + a = 0 neither touches nor intersects the coordinates
axes. If the point (1, 4) lies inside the circle, then
(A) 25 < a < 29 (B) 0 < a < 29
(C) a > 29 (D) none of these
(iii) If the point ( , ), with integral values of , , lies in the interior of the quadrilateral
enclosed by the lines x = 0, y = 0, x + 2y  4 =0, 2x + 3y  12 = 0, then one possible
position of P is at
(A) (2, 1) (B) (2, 3)
(C) (2, 2) (D) (4, 2)

XI. Concept: From a point P (h, k), in general, three normals can be drawn to the parabola
y2= 4ax. If t1, t2, t3 are the parameters associated with the feet of these normals, then t1, t2, t3 are
the roots of the equation at3 + (2a  h)t  k = 0. Moreover, from any point on the line x =  a, two
perpendicular tangents can be drawn to the parabola.

Illustration :  
If the tangents at the feet Q at12 , 2at1 and R at 22 , 2at 2   to the parabola meet
on the line x =  a, then t1, t2 are the roots of the equation
2
(A) t  t3t + 1 = 0 (B) t2 + t3t + 1 = 0
2
(C) t  t3t  1 = 0 (D) t2 + t3t  1 = 0

Solution: From the given equation in t, we find that


t1 + t2 + t3 = 0  t1 + t2 =  t3 .
1 1 1
The slope of the tangent at Q is and that of at P is   1
t1 t2 t1t 2
 t1t2 = 1.
Hence (D) is the correct answer.

FIITJEE Ltd., ICES House, 29-A, Kalu Sarai, Sarvapriya Vihar, New Delhi -110016, Ph 26515949, 26569493, Fax 26513942
FIITJEE Ltd. Material Provided by - Material Point Available on - Learnaf.com

P-57-P8-CBSE-MA-83 Pinnacle Study Package

Exercise 11
(i).   
If the feet Q at12 , 2at1 and R at 22 , 2at 2  are the ends of a focal chord of the
parabola, then the locus of P (h, k) is
(A) y2 = a (x  2a) (B) y2 = a (x  a)
2
(C) y = a (x  3a) (D) y2 = 3a (x  a)
(ii). If P (h, k) is a vertex of the square comprising normals to the parabola from P and
tangents from the directrix, then (h, k) is the same as
(A) (a, 0) (B) (2a, 0)
(C) (3a, 0) (D) (4a, 0)
(iii). Tangents are drawn from the directrix to the parabola. The normals at the points of
contact meet at the point P (h, k) to form a square. The radius of the circle
circumscribing this square is
(A) a (B) 2a
(C) 4a (D) a/2

FIITJEE Ltd., ICES House, 29-A, Kalu Sarai, Sarvapriya Vihar, New Delhi -110016, Ph 26515949, 26569493, Fax 26513942
FIITJEE Ltd. Material Provided by - Material Point Available on - Learnaf.com
M ast er A ssignm ent s P-57-P8-CBSE-MA-84

ANSWERS TO EXERCISES

Exercise 1
(i) (C) (ii) (B) (iii) (A)

Exercise 2
(i). D (ii). A (iii). A

Exercise 3
(i) C (ii) C (iii) C

Exercise 4
(i). D (ii). B (ii) A

Exercise 5
(i) (C) (ii). B (iii). C

(iv). B

Exercise 6
(i). C (ii) D (iii) A

Exercise 7
(i). C (ii) B (iii) A

Exercise 8
(i). B (ii) A (iii) B

Exercise 9
(i). A (ii) C (iii) B

Exercise 10
(i). C (ii) A (iii) C

Exercise 11
(i). C (ii) C (iii) B

FIITJEE Ltd., ICES House, 29-A, Kalu Sarai, Sarvapriya Vihar, New Delhi -110016, Ph 26515949, 26569493, Fax 26513942
----------------------------------------------------------------------------------------------------------------------------------------------

------------------------------------------------------------------------------------------------------------------------------------------------
Thank you for your love and support, we hope you are always being happy and get success in your life, we
are happy to see you again.

Regards from Learnaf team


Click on very top right corner for download and other more options.

S-ar putea să vă placă și